Preguntas de Endocrino

42
CTO Medicina • C/Nuñez de Balboa 115 • 28006 Madrid • Tfno.: 91 782 43 30/33/34 • E-mail: [email protected] • www. grupocto.es 1 Test 3V Endocrinología y Metabolismo CTO MEDICINA T EMA 1. FISIOLOGÍA DEL SISTEMA ENDOCRINO 1 Pon ejemplos de hormonas de cada uno de estos tipos de receptores - Receptores de membrana: la gran mayoría de hormonas peptídicas y proteicas (PTH, ACTH, TSH, glucagón, insulina, GH, prolactina…) - Receptores citosólicos: hormonas esteroideas - Receptores nucleares: hormonas tiroideas 2 Única hormona hipofisaria que presenta un control hipotalámico fundamentalmente inhibitorio Prolactina 3 Principal mediador que inhibe la secreción de prolactina Dopamina 4 La oxitocina y la vasopresina son hormonas que se sintetizan en…y se almacenan en… Neuronas de los núcleos supraóptico y paraventricular del hipotálamo…… lóbulo posterior de la hipófisis 5 Ordena de mayor a menor las células de la hipófisis Células somatotróficas (50%), células corticotróficas (20%), células lactotróficas (10-30%), células gonadotróficas (10%) y células tirotróficas (5%) 6 Papel fundamental de la GH Estimular el crecimiento lineal normal 7 Principales mediadores de la acción de la GH sobre el crecimiento Las somatomedinas 8 ¿Cuál es la más importante? La somatomedina C o IGF-1 9 La IGF-1 se produce en… El hígado 10 La IGF-1 es responsable del crecimiento, ¿en qué etapa? En la etapa puberal de la vida 11 ¿Cuál es la primera hormona que se altera cuando existe una lesión hipotalámica o panhipopituitarismo postcirugía o radioterapia? La GH 12 Principal acción de la ACTH Liberación de cortisol a partir de la corteza suprarrenal 13 Alteraciones hormonales que marcan el desarrollo puberal Aumento de la sensibilidad a GNRH y comienzo de secreción pulsátil de LH 14 Principal función de la prolactina Estimular la lactancia 15 Hormonas encargadas de potenciar el crecimiento ductal mamario durante el embarazo Los estrógenos 16 Suceso que determina el comienzo de la lactancia tras el parto y no durante el embarazo El descenso de los estrógenos tras el parto

Transcript of Preguntas de Endocrino

Page 1: Preguntas de Endocrino

CTO Medicina • C/Nuñez de Balboa 115 • 28006 Madrid • Tfno.: 91 782 43 30/33/34 • E-mail: [email protected] • www. grupocto.es 1

Test 3VEndocrinologíay Metabolismo

CTO MEDICINA

TEMA 1. FISIOLOGÍA DEL SISTEMA ENDOCRINO

1 Pon ejemplos de hormonas de cada uno de estos tipos de receptores

- Receptores de membrana: la gran mayoría de hormonas peptídicas y proteicas (PTH, ACTH, TSH, glucagón, insulina, GH, prolactina…)- Receptores citosólicos: hormonas esteroideas- Receptores nucleares: hormonas tiroideas

2Única hormona hipofi saria que presenta un control hipotalámico fundamentalmente inhibitorio

Prolactina

3 Principal mediador que inhibe la secreción de prolactina Dopamina

4 La oxitocina y la vasopresina son hormonas que se sintetizan en…y se almacenan en…

Neuronas de los núcleos supraóptico y paraventricular del hipotálamo…… lóbulo posterior de la hipófi sis

5 Ordena de mayor a menor las células de la hipófi sisCélulas somatotrófi cas (50%), células corticotrófi cas (20%), células lactotrófi cas (10-30%), células gonadotrófi cas (10%) y células tirotrófi cas (5%)

6 Papel fundamental de la GH Estimular el crecimiento lineal normal

7 Principales mediadores de la acción de la GH sobre el crecimiento

Las somatomedinas

8 ¿Cuál es la más importante? La somatomedina C o IGF-1

9 La IGF-1 se produce en… El hígado

10 La IGF-1 es responsable del crecimiento, ¿en qué etapa? En la etapa puberal de la vida

11¿Cuál es la primera hormona que se altera cuando existe una lesión hipotalámica o panhipopituitarismo postcirugía o radioterapia?

La GH

12 Principal acción de la ACTH Liberación de cortisol a partir de la corteza suprarrenal

13 Alteraciones hormonales que marcan el desarrollo puberalAumento de la sensibilidad a GNRH y comienzo de secreción pulsátil de LH

14 Principal función de la prolactina Estimular la lactancia

15 Hormonas encargadas de potenciar el crecimiento ductal mamario durante el embarazo

Los estrógenos

16Suceso que determina el comienzo de la lactancia tras el parto y no durante el embarazo

El descenso de los estrógenos tras el parto

Page 2: Preguntas de Endocrino

CTO Medicina • C/Núñez de Balboa 115 • 28006 Madrid • Tfno.: 91 782 43 30/33/34 • E-mail: [email protected] • www. grupocto.es 2

Test 3V Endocrinología y MetabolismoCTO MEDICINA

TEMA 1. FISIOLOGÍA DEL SISTEMA ENDOCRINO

17¿Cuál es la hormona hipofi saria cuya concentración aumenta cuando se destruye el hipotálamo o se secciona el tallo hipofi sario?

La prolactina

18 Factores que estimulan la producción de prolactinaEl estrés, la lesión en la pared torácica, la succión del pezón, los opiáceos, los neurolépticos, los estrógenos…

19El principal factor hipotalámico que regula la liberación de TSH es…, y su principal estímulo inhibidor son….

La TRH…..las hormonas tiroideas

20 ¿Cuáles son las hormonas que tienen una gran similitud estructural por compartir la subunidad alfa?

LH, FSH, TSH y hCG

21 ¿Qué hormona tiroidea se sintetiza únicamente en la tiroides? La T4

22 ¿Dónde se produce fundamentalmente la T3? En tejidos periféricos a partir de la T4

23¿Cuál de las hormonas tiroideas tiene mayor acción sobre los tejidos y es responsable de casi todos sus efectos?

La T3

24 Acción fundamental de la vasopresina Controlar la conservación de agua

25 Acción fundamental de la oxitocina Estimula las contracciones uterinas y la eyección de la leche

26 Principal regulador de la liberación de hormona antidiurética El aumento de la osmolaridad plasmática

27 Estímulos que aumentan la liberación de hormona antidiurética

Aumento de la osmolaridad plasmática, disminución de volumen plasmático, disminución de la presión arterial,fármacos como la nicotina, morfi na, clofi brato, antidepresivos tricíclicos…; la hipertermia, la hipoxia, la hipoglucemia y otros

28 Fármacos que inhiben la liberación de ADH Etanol, naloxona, difenilhidantoína y clorpromacina

29 ¿Cuál es el enzima fundamental para la síntesis de hormonas tiroideas?

La peroxidasa tiroidea

30 Las hormonas tiroideas viajan en sangre unidas aGlobulina fi jadora de hormonas tiroideas (TBG), a la transtiretina o prealbúmina (TBPA) y a la albúmina

31¿Cuál es el estado hormonal de la hipertiroxinemia eutiroidea?

T4 total elevada y T4 libre normal

32Fármacos capaces de disminuir la conversión periférica de T4 en T3

Amiodarona, dexametasona, propranolol, contrastes yodados, propiltiouracilo

33Une cada tipo de hormona suprarrenal con su lugar de síntesis

- Mineralcorticoides (aldosterona): capa glomerular de la corteza suprarrenal- Glucocorticoides (cortisol): capa fascículo-reticular de la corteza- Andrógenos (DHEA): capa fascículo-reticular de la corteza- Adrenalina y noradrenalina: médula suprarrenal

34 La liberación máxima de ACTH y de cortisol tiene lugar fundamentalmente

Por la mañana

35 El cortisol en sangre circula unido aTranscortina o globulina transportadora de cortisol (CBG) y a la albúmina

Page 3: Preguntas de Endocrino

CTO Medicina • C/Núñez de Balboa 115 • 28006 Madrid • Tfno.: 91 782 43 30/33/34 • E-mail: [email protected] • www. grupocto.es 3

Test 3V Endocrinología y MetabolismoCTO MEDICINA

TEMA 1. FISIOLOGÍA DEL SISTEMA ENDOCRINO

36 La liberación de cortisol está regulada fundamentalmente por… y ejerce su acción vale de receptores…

La ACTH ….. tipo II

37La secreción de mineralcorticoides está regulada fundamentalmente por…y ejercen su acción fundamentalmente a través de receptores

El sistema renina-angiotensina-aldosterona …tipo I

38 Lugar de producción de la renina Células yuxtaglomerulares renales

39 El angiotensinógeno se sintetiza en El hígado

40La enzima conversora de la angiotensina se encuentra fundamentalmente en

En endotelio vascular del pulmón

41 Factores que estimulan la liberación de reninaLa disminución de la perfusión renal, la disminución de cloro en el túbulo distal inicial, el sistema nervioso simpático, la disminución de potasio

42 Acciones fundamentales de la angiotensina II

Aumenta la reabsorción de sal en el túbulo proximal, favorece la reabsorción renal, estimula la liberación de aldosterona, estimula el centro de la sed, estimula liberación de ADH y produce vasoconstricción

43 Acciones fundamentales de la aldosteronaRegula el volumen del líquido extracelular, aumenta la reabsorción de sodio y la eliminación de potasio e hidrogeniones en la orina

44 Mecanismos que controlan la liberación de la aldosteronaEl sistema renina-angiotensina, el potasio y la ACTH (importancia secundaria)

45Diferencia fundamental entre la insufi ciencia suprarrenal primaria y secundaria

Primaria: défi cit de glucocorticoides y mineralcorticoides, con alteraciones en el equilibrio ácido-base y en el potasio. Secundaria: se produce por défi cit de ACTH, niveles de aldosterona normales, no se altera potasio ni equilibrio ácido-base

46Las altas dosis de glucocorticoides pueden tener también efecto

Mineralcorticoide

47 Andrógenos suprarrenalesDehidroepiandrosterona (DHEA) y sulfato de dehidroepiandrosterona (DHEAs)

48Principal órgano productor de estrógenos en mujer no gestante

Ovario

49 Principal productor de estrógenos en la mujer gestante Placenta

50 ¿Cuál es el principal estrógeno ovárico? El 17-betaestradiol

51 ¿Dónde se produce la progesterona? En el cuerpo lúteo y en la placenta

52 Principal forma de circulación en la sangre de andrógenos y estrógenos

Unidos a la globulina transportadora de hormonas sexuales (SHBG)

53Principal hormona implicada en el metabolismo hidrocarbonato: …. Se sintetiza en…, su precursor es…, que se escinde en…:

La insulina… células beta de los islotes pancreáticos… la proinsulina… insulina y péptido C

Page 4: Preguntas de Endocrino

CTO Medicina • C/Núñez de Balboa 115 • 28006 Madrid • Tfno.: 91 782 43 30/33/34 • E-mail: [email protected] • www. grupocto.es 4

Test 3V Endocrinología y MetabolismoCTO MEDICINA

TEMA 1. FISIOLOGÍA DEL SISTEMA ENDOCRINO

54 El glucagón se sintetiza en Células alfa de los islotes pancreáticos

55 Principales hormonas contrainsularesGlucagón, catecolaminas, estrógenos, gestágenos, GH y esteroides suprarrenales

56El metabolismo fosfocálcico está regulado fundamentalmente por

PTH, vitamina D y calcitonina

57 La secreción de PTH se produce en …. y está regulada fundamentalmente por

Glándulas paratiroides ….. la fracción de calcio libre en el líquido extracelular

58 Hormonas que se encargan de elevar la calcemia La PTH y la vitamina D

59 Principales fuentes de vitamina D La piel y la dieta

60Pasos necesarios para conseguir una vitamina D en su forma activa

Hidroxilación en posición 25 en el hígado, e hidroxilación en posición 1 en el riñón

61 Principales acciones de la vitamina DAumenta la absorción de calcio y fósforo a nivel intestinal y facilita resorción ósea en el hueso

62 Acciones de la PTHEn el hueso favorece la resorción ósea y en el riñón aumentala fosfaturia, aumenta la reabsorción de calcio y favorece la activación de la vitamina D

63 La calcitonina es sintetiza en Células C o parafoliculares de la tiroides

64 Acciones fundamentales de la calcitoninaInhibe la resorción ósea y disminuye la reabsorción tubular de calcio y fósforo

65 Precursor de las prostaglandinas El ácido araquidónico

66 La síntesis de prostaglandinas requiere de un enzima, ¿cuál? La ciclooxigenasa

67 Principales efectos del tromboxano A2 Vasoconstrictor y antiagregante plaquetario

68 El principal péptido natriurético es….y se sintetiza en… El péptido atrial natriurético…. el tejido auricular

69 El centro del hambre está situado en… y es regulado fundamentalmente por… que se sintetiza en…

El hipotálamo…. el neuropéptido Y…… el núcleo arcuato del hipotálamo

70 La grelina es un péptido secretado en… cuya función parece que es…

El estómago …..estimulación del apetito tras unas horas después de la ingesta

71 Relaciona cada tipo de lipoproteína con los lípidos que transporta

- Quilomicrones y partículas residuales: triglicéridos y colesterol ingerido con la dieta- VLDL: triglicéridos endógenos sintetizados por el hígado- IDL: ésteres de colesterol y triglicéridos- LDL: ésteres de colesterol- HDL: ésteres de colesterol

Page 5: Preguntas de Endocrino

CTO Medicina • C/Núñez de Balboa 115 • 28006 Madrid • Tfno.: 91 782 43 30/33/34 • E-mail: [email protected] • www. grupocto.es 5

Test 3V Endocrinología y MetabolismoCTO MEDICINA

TEMA 1. FISIOLOGÍA DEL SISTEMA ENDOCRINO

72 Relaciona cada lipoproteína con su apoproteína correspondiente

- Quilomicrones: apoproteína CII, CIII, AI, AII, B48, E- Quilomicrones residuales : apoproteína B48 y E- VLDL: apoB100, CI, CII; CIII, E- IDL : apoB100, CIII, E- LDL : B100- HDL : AI, AII

73¿Qué lipoproteína es la encargada de transportar el colesterol necesario a las glándulas suprarrenales para la síntesis de hormonas?

Las LDL

74 ¿Cuál es el principal modulador de la síntesis de colesterol y de receptores LDL?

El colesterol libre intracelular

TEMA 2. ENFERMEDADES DE LA HIPÓFISIS Y EL HIPOTÁLAMO

1 Causa más frecuente de hiperprolactinemia Fármacos

2Manifestaciones clínicas fundamentales de la hiperprolactinemia en mujeres

Oligomenorrea o amenorrea, infertilidad por ciclos anovulatorios y galactorrea

3 Manifestaciones clínicas de la hiperprolactinemia en el varón Disminución de la líbido, impotencia e infertilidad

4Ante una hiperprolactinemia hay que descartar una serie de causas antes del diagnóstico de enfermedad hipofi saria o hipotalámica, ¿cuáles?

Embarazo, puerperio, cirrosis, estados postcríticos, ingesta de medicamentos, hipotiroidismo o insufi ciencia suprarrenal

5Paciente varón que presenta ginecomastia, ¿qué es lo primero que se debe descartar?

Existencia de tumor productor de estrógenos o beta-hCG

6Una paciente de 30 años de edad consulta por oligomenorrea e imposibilidad para quedarse embarazada tras dos años. ¿Qué determinación habría que hacer en primer lugar?

Niveles de prolactina

7 Niveles séricos de prolactina por encima de 250 ug/l son diagnósticos de

Adenoma hipofi sario productor de PRL

8 ¿Cuáles son los adenomas hipofi sarios más frecuentes? Los prolactinomas

9 Se habla de microadenomas hipofi sarios si el tamaño es… Menor de 10 mm

10

Paciente de 50 años, con una cifra de prolactina sérica de 75 ug/l, en la RM de la región selar se observa una lesión de 15 mm, ¿qué sería lo que habría que sospechar en primer lugar?

Adenoma no funcional con hiperprolactinemia por compresión del tallo hipofi sario

11 Los prolactinomas más frecuentes son Los microprolactinomas

12Los microprolactinomas son más frecuentes en… y los macroprolactinomas en…

Mujeres….. varones

13Una mujer de 33 años de edad en tratamiento con anticonceptivos orales desde hace tres años comienza con galactorrea, ¿cuál sería el siguiente paso a seguir?

Medir niveles de PRL

Page 6: Preguntas de Endocrino

CTO Medicina • C/Núñez de Balboa 115 • 28006 Madrid • Tfno.: 91 782 43 30/33/34 • E-mail: [email protected] • www. grupocto.es 6

Test 3V Endocrinología y MetabolismoCTO MEDICINA

TEMA 2. ENFERMEDADES DE LA HIPÓFISIS Y EL HIPOTÁLAMO

14Tratamiento de primera elección en la hiperprolactinemia: agonistas dopaminérgicos clásicos como bromocriptina, lisuride y pergolide, o modernos

Como quinagolida y cabergolina

15

Paciente varón de 48 años de edad, que presenta disminución de la líbido e impotencia desde hace dos meses, alteraciones en el campo visual y una cifra de prolactina de 200 ug/l, ¿cuál sería el tratamiento inicial de elección?

Agonistas dopaminérgicos como la bromocriptina

16Si tras el tratamiento médico con cabergolina no se produce mejoría en el campo visual, ¿qué opción es la más correcta?

Resección transesfenoidal del tumor

17Paciente de 28 años en tratamiento por un microprolactino-ma, se queda embarazada, ¿qué se debe hacer?

Suspender la medicación y medir concentraciones de PRL periódicamente (si superasen los 400 ug/l se considera que ha existido crecimiento tumoral y se reinicia tratamiento con bromocriptina)

18Los microprolactinomas se tratan si…. y los macroprolactinomas se tratan…

Producen síntomas….. siempre

19¿Cuál es la primera manifestación clínica que aparece tras un infarto hipofi sario postparto o síndrome de Sheehan?

Incapacidad para la lactancia

20 El exceso de hormona de crecimiento (GH) se manifi esta porGigantismo, si se produce antes del cierre de las epífi sis, oacromegalia, si se produce tras el cierre de las epífi sis

21 La mayoría de los pacientes acromegálicos presentan Un adenoma hipofi sario, más frecuentemente un macroadenoma

22

Varón de 44 años de edad que en los últimos dos meses se encuentra cansado, débil, con aumento de la sudoración, y ha observado que no le cabe en el dedo su anillo de casado; que las manos las tiene húmedas y pastosas y la voz cavernosa. La analítica demuestra una glucosa elevada, ¿qué habrá que sospechar en primer lugar?

Acromegalia por macroadenoma hipofi sario

23 Si en el caso anterior se encontrara también que el paciente presenta hipercalcemia, ¿qué habría que descartar?

La existencia de un hiperparatiroidismo primario dentro de un MEN1

24¿Cuáles serían las pruebas diagnósticas de elección en el caso del paciente anterior?

Determinación de los niveles de IGF-1 y la supresión de la secreción de GH con la sobrecarga de glucosa

25

Si obtenemos los resultados siguientes: niveles de GH a las dos horas de la sobrecarga de 2,5 ug/L y niveles de IGF-1 elevados para el sexo y edad del paciente, ¿cuál sería el siguiente paso a realizar?

Prueba de imagen: RM y TAC craneal y estudios del campo visual

26 Tratamiento de elección en los macroadenomas productores de GH

Cirugía transesfenoidal

27 El tratamiento médico fundamental de la acromegalia se basa en

Análogos de la somatostatina: octreótido o lanreótido

28 Efectos secundarios fundamentales de los análogos de la somatostatina

Esteatorrea y dolor abdominal, y a largo plazo, colelitiasis

29 Tratamiento médico de segunda elección en la acromegalia Pegvisomant (antagonista del receptor de GH)

Page 7: Preguntas de Endocrino

CTO Medicina • C/Núñez de Balboa 115 • 28006 Madrid • Tfno.: 91 782 43 30/33/34 • E-mail: [email protected] • www. grupocto.es 7

Test 3V Endocrinología y MetabolismoCTO MEDICINA

TEMA 2. ENFERMEDADES DE LA HIPÓFISIS Y EL HIPOTÁLAMO

30¿Por qué es imprescindible tratar a los pacientes acromegálicos?

Porque estos pacientes tienen mayor riesgo de mortalidad por causas cardiovasculares y tumores y, al tratarlos, esta mortalidad se iguala a la población general

31 El défi cit congénito de GH se manifi esta a partir de los… 6-12 meses de edad

32 La talla y el peso al nacer en los niños con défi cit de GH son Normales

33 Clínica característica del défi cit de GH

Detención del crecimiento, edad ósea retrasada, obesidad troncular, frente amplia, raíz nasal hundida, mejillas redondeadas (aspecto de muñeco), dentición retrasada, y en el varón puede existir micropene

34 Diagnóstico fundamental del défi cit de GHPruebas de estimulación de GH, farmacológicas (clonidina, arginina, hipoglucemia insulínica) o fi siológicas (ejercicio)

35 Si el valor de GH obtenido en las pruebas de estimulación es inferior a 5 ug/l se considera que existe…

Défi cit de GH

36Diferencia fundamental entre un défi cit de GH y un síndrome de resistencia a GH

En el défi cit de GH, niveles reducidos de GH y de IGF-1; en el síndrome de resistencia a GH, niveles reducidos de IGF-1 y elevados de GH

37 Tratamiento fundamental en un niño con défi cit de GH GH sintética

38Tratamiento fundamental en un niño con enanismo de Laron (insensibilidad a la GH)

IGF-1 recombinante

39 Causa más frecuente de défi cit de GH en el adulto Tumores hipofi sarios y paraselares

40 Clínica típica del défi cit de GH en adultoAlteración de la composición corporal, aumento de la grasa, disminución de la masa muscular, disminución de la fuerza física, alteraciones psicológicas y de la calidad de vida

41 Diagnóstico del défi cit de GHTest de estimulación (hipoglucemia insulínica) y medición de IGF-1 basal

42 Objetivos del tratamiento con GHMejoría en la calidad de vida, corrección de las alteraciones de la composición corporal y metabólicas, recuperación de la masa ósea y muscular…

43 Contraindicaciones del tratamiento con GHRetinopatía diabética proliferativa, cuando existen signos de malignidad activa, e hipertensión intracraneal

44Características bioquímicas del hipogonadismo hipogonadotrófi co o central

Niveles séricos bajos de testosterona y FSH y LH disminuidas

45El défi cit aislado de gonadotropinas por defecto en la síntesis de GNRH (de causa idiopática) se denomina

Síndrome de Kallman

46 Diagnóstico de síndrome de KallmanLa administración de GNRH sintética de forma pulsátil restaura la respuesta de gonadotropinas

47 Defectos adquiridos de la producción de GNRHLa hiperprolactinemia, la anorexia nerviosa, la desnutrición, el ejercicio físico intenso y el estrés

Page 8: Preguntas de Endocrino

CTO Medicina • C/Núñez de Balboa 115 • 28006 Madrid • Tfno.: 91 782 43 30/33/34 • E-mail: [email protected] • www. grupocto.es 8

Test 3V Endocrinología y MetabolismoCTO MEDICINA

TEMA 2. ENFERMEDADES DE LA HIPÓFISIS Y EL HIPOTÁLAMO

48Características del hipotiroidismo central que permiten diferenciarlo del hipotiroidismo primario

No existe bocio, no presentan elevación de colesterol y se suele asociar al défi cit de otras hormonas hipofi sarias. En la analítica: niveles bajos de T4 y bajos de TSH

49Para el control del tratamiento del hipotiroidismo hipofi sario, ¿cuál es la determinación más útil?

Los niveles de T4 o T3 libres

50 Características bioquímicas del hipertiroidismo hipofi sario Niveles elevados de T4 con niveles de TSH normales o elevados

51 ¿Cómo se diferencia el hipertiroidismo hipofi sario de la resistencia periférica a hormonas tiroideas?

Por la existencia de tumor hipofi sario y un cociente TSH alfa/TSH > 1 en caso de un hipertiroidismo hipofi sario

52Diferencias entre la insufi ciencia suprarrenal y el défi cit de secreción de ACTH

En el défi cit de ACTH no existe hiperpigmentación ni hiperpotasemia (porque no se altera la vía de los mineralcorticoides)

53 Ordena de mayor a menor frecuencia los tumores secretores de hipófi sis

- Prolactinomas- Tumores productores de GH- Adenomas secretores de ACTH- Adenomas productores de gonadotropinas- Adenomas productores de TSH

54 Manifestaciones locales de los adenomas hipofi sariosDefectos campimétricos, parálisis oculomotoras, obstrucción y compresión de la arteria carótida, cefalea…

55Defecto campimétrico más frecuente en un adenoma de hipófi sis

Hemianopsia bitemporal

56 Actitud ante la sospecha de un adenoma hipofi sarioDeterminaciones hormonales, para aclarar si se trata de un tumor secretor y descartar defi ciencias hormonales por destrucción de la hipófi sis, y pruebas de imagen: TAC, RM

57 ¿Qué es la apoplejía hipofi saria? Es el infarto hemorrágico agudo de un adenoma hipofi sario

58 Clínica de una apoplejía hipofi sariaCefalea, náuseas, vómitos, disminución del nivel de conciencia, síntomas meníngeos, oftalmoplejía y alteraciones pupilares

59 Secuela más frecuente tras una apoplejía hipofi saria Panhipopituitarismo

60 Tratamiento fundamental de la apoplejía hipofi saria Descompresión de la hipófi sis por vía transesfenoidal

61 Complicaciones fundamentales de la cirugía transesfenoidal de un microadenoma

Rinorrea de líquido cefalorraquídeo, parálisis del tercer par y pérdida de visión

62 Complicaciones fundamentales de la cirugía transesfenoidal de un macroadenoma

Hipopituitarismo, diabetes insípida transitoria y permanente, rinorrea de LCR y pérdida visual

63Secuencia en que se produce la pérdida hormonal en caso de un hipopituitarismo de causa aguda

ACTH, LH/FSH, TSH

64Secuencia en que se produce la pérdida hormonal en caso de un hipopituitarismo de causa crónica, por ejemplo, por crecimiento de un adenoma

GH, LH/FSH, TSH y ACTH

65Se habla de síndrome de Sheehan cuando existe… y es típico de…

Necrosis hipofi saria….. mujeres embarazadas, cuando el parto se complica con hemorragia intensa e hipotensión

Page 9: Preguntas de Endocrino

CTO Medicina • C/Núñez de Balboa 115 • 28006 Madrid • Tfno.: 91 782 43 30/33/34 • E-mail: [email protected] • www. grupocto.es 9

Test 3V Endocrinología y MetabolismoCTO MEDICINA

TEMA 2. ENFERMEDADES DE LA HIPÓFISIS Y EL HIPOTÁLAMO

66 Diagnóstico del hipopituitarismoEstudio de imagen (RM y TC), estudio oftalmológico y campimétrico, determinación de niveles basales hormonales y pruebas dinámicas para valorar reserva hipofi saria

67 Tratamiento del hipopituitarismo

Administrar hormonas defi citarias, comenzando por sustituir los glucocorticoides antes que las hormonas tiroideas para evitar crisis suprarrenal (GH, hidrocortisona oral o iv, levotiroxina y esteroides gonadales, LH y FSH o GNRH en bomba con pulsos)

68El síndrome de silla turca vacía primaria es típico de… función hipofi saria suele ser….

Mujeres obesas, multíparas e hipertensas…. normal, pero puede existir hiperprolactinemia por compresión del tallo

69 Característica fundamental de la diabetes insípidaLiberación por el organismo de grandes cantidades de orina diluida (poliuria hipotónica)

70 Causas de diabetes insípida- Falta de liberación de ADH (diabetes insípida central).- Ausencia de respuesta del riñón a la ADH (diabetes insípida nefrogénica)

71 Etiología más frecuente de la diabetes insípida centralIdiopática, traumatismos craneoencefálicos y tumores cerebrales o hipofi sarios

72 En síndrome de Wolfram se caracteriza porDiabetes mellitus, diabetes insípida central, atrofi a óptica y sordera neurosensorial

73 Causas más frecuentes de diabetes insípida nefrogénicaAdquiridas: hipercalcemia, hipopotasemia, enfermedades tubulointersticiales y fármacos como el litio

74 Clínica fundamental de la diabetes insípidaPoliuria persistente, polidipsia y sed excesiva. Orina con baja densidad (<1010) y osmolaridad disminuida (< 300 mosm/kg), con osmolaridad plasmática elevada (>290 mosm/kg)

75Paciente con osmolaridad plasmática de 295 mosm/kg y osmolaridad urinaria de 290 mosm/kg, ¿cuál es el siguiente paso a realizar?

Prueba terapéutica con vasopresina o desmopresina.

76Paciente con osmolaridad plasmática de 285 mosm/kg y osmolaridad urinaria de 290 mosm/kg, ¿cuál es la primera prueba que debería realizarse?

La prueba de deshidratación o prueba de la sed (test de Miller)…..

77 Tratamiento fundamental de la diabetes insípida centralSustitución hormonal con diferentes preparados por vía intranasal o parenteral

78 En pacientes con diabetes insípida, que estén hipotensos o con sintomatología neurológica el principal tratamiento es

Administración de suero salino fi siológico al 0,9% iv

79Tratamiento de los pacientes con diabetes insípida nefrogénica

Restricción de sal, y diuréticos que aumenten la natriuresis, como las tiacidas, y AINEs

80Característica fundamental de la secreción inadecuada de ADH

Presencia de hiponatremia secundaria a la retención de agua libre por secreción de ADH inapropiadamente elevada

81 Causas de secreción inadecuada de ADH

- Neoplasias: microcítico de pulmón, páncreas, timoma…- Enfermedades pulmonares no malignas: tuberculosis, absceso, EPOC- Alteraciones del SNC: encefalitis, meningitis…- Fármacos: opiáceos, nicotina, carbamacepina, ciclofosfamida…

Page 10: Preguntas de Endocrino

CTO Medicina • C/Núñez de Balboa 115 • 28006 Madrid • Tfno.: 91 782 43 30/33/34 • E-mail: [email protected] • www. grupocto.es 10

Test 3V Endocrinología y MetabolismoCTO MEDICINA

TEMA 2. ENFERMEDADES DE LA HIPÓFISIS Y EL HIPOTÁLAMO

82Criterios diagnósticos de síndrome de secreción inadecuada de ADH

- Hiponatremia < 135 mmol/l- Hipoosmolaridad plasmática < 275 mosm/kg- Orina no diluida al máximo > 100 mosm/kg- No edemas- No depleción de volumen (TA normal)- Exclusión de hipotiroidismo e insufi ciencia suprarrenal

83 Pruebas diagnósticas para detectar una secreción inadecuada de ADH

Prueba de la sobrecarga hídrica: es normal si a las 5 horas se elimina el 80% del líquido ingerido

84 Contraindicación para la prueba de sobrecarga hídrica Sodio plasmático < 125 mmol/l

85Tratamiento fundamental de una hiponatremia aguda con alteraciones del SNC, irritabilidad, confusión, etc…… y sodio < 125

Suero salino hipertónico en bomba de perfusión y furosemida

86Tratamiento fundamental de una hiponatremia leve, con sodio entre 130 y 135 y asintomática

Restricción hídrica 500-1000 cc al día, aportes de sal, dosis bajas de furosemida, y fármacos como la demeclociclina o el litio

TEMA 3. ENFERMEDADES DE TIROIDES

1

Paciente sin antecedentes de patología tiroidea que ingresa en la Unidad de Cuidados Intensivos de un hospital en situación de sepsis severa. Niveles de THS y T4 libre normales con T3 baja, diagnóstico más probable

Síndrome de enfermedad sistémica no tiroidea

2Hallazgos de laboratorio característicos del síndrome del eutiroideo enfermo

Disminución de T3 libre, T4 total y TSH pueden ser normales o bajas, y T3R aumentada

3 Tratamiento actual del síndrome del eutiroideo enfermo No requiere tratamiento

4 Causas más frecuentes de bocio simple

Défi cit de yodo en la dieta, ingesta de bociógenos (litio, antitiroideos, compuestos yodados), enfermedades infl amatorias o infi ltrativas, tumores o defectos en la síntesis de hormonas tiroideas

5 Patrón hormonal tiroideo necesario para poder hablar de bocio simple

Función tiroidea normal (es decir, niveles de T4, T3 y TSH dentro de la normalidad)

6 Clínica del síndrome de Pendred Bocio, sordera neurógena y eutiroidismo o hipotiroidismo leve

7Clínica característica del bocio simple: síntomas compresivos locales

Disfagia, molestias locales, congestión facial al elevar los brazos (signo de Pemberton)…

8Sospecha clínica fundamental ante un bocio con afectación de los nervios laríngeos recurrentes

Carcinoma de tiroides

9 Diagnóstico de bocio simpleNiveles hormonales normales y gammagrafía normocaptante o levemente hipercaptante

10 Tratamiento del bocio simple con síntomas compresivos Tiroidectomía subtotal

11 Tratamiento del bocio simple sin síntomas compresivosLevotiroxina oral en dosis supresoras, administración de I131 sal yodada o vigilancia clínica

Page 11: Preguntas de Endocrino

CTO Medicina • C/Núñez de Balboa 115 • 28006 Madrid • Tfno.: 91 782 43 30/33/34 • E-mail: [email protected] • www. grupocto.es 11

Test 3V Endocrinología y MetabolismoCTO MEDICINA

TEMA 3. ENFERMEDADES DE TIROIDES

12Efectos secundarios de la levotiroxina en dosis supresoras a largo plazo

Osteoporosis y cardiopatía

13 ¿En qué consiste el efecto Jod-Basedow?

En la posibilidad de que se desencadene una crisis tirotóxica tras administrar dosis altas de yodo en pacientes con bocio o que presentan algún nódulo con capacidad de funcionamiento autónomo

14 Causas más frecuentes de hipotiroidismo en niñosHipoplasia o aplasia de tiroides, défi cit congénito de síntesis de hormonas tiroideas, hipopituitarismo, tiroiditis de Hashimoto, défi cit grave de yodo

15 Causas más frecuentes de hipotiroidismo en adultosTiroiditis de Hashimoto, ablación postradioyodo, postcirugía, postradioterapia externa; hipotiroidismo idiopático autoinmune

16 Causa más frecuente de hipotiroidismo a nivel mundial Défi cit de yodo

17Causa más frecuente de hipotiroidismo en países desarrollados

Hipotiroidismo autoinmune y el hipotiroidismo postradioyodo y postcirugía

18Recién nacido de tres días de vida en el que se aprecia una coloración amarillenta de piel y mucosas, llanto ronco, somnolencia y estreñimiento, ¿qué debe sospecharse?

Hipotiroidismo congénito

19 Cuadro clínico característico del cretinismoTalla baja, rasgos toscos, lengua prominente, escasez de vello, sequedad de piel, retraso en la edad ósea y retraso de la dentición, alteración del desarrollo mental

20 Clínica típica del hipotiroidismo

Fatiga, letargia, estreñimiento, intolerancia al frío, rigidez y contractura muscular, síndrome del túnel carpiano, trastornos menstruales, pérdida de apetito, demencia, piel seca y áspera, alopecia, pérdida de memoria, bradipsiquia, aumento de peso

21 Prueba más útil para el diagnóstico de hipotiroidismo Determinación de TSH

22 Diferencias entre hipotiroidismo primario y secundarioTSH aumentada en hipotiroidismo primario y normal o indetectable en hipotiroidismo hipofi sario

23¿En qué tipo de hipotiroidismo es característico el aumento del colesterol sérico?

En el hipotiroidismo primario

24 Situación hormonal que nos permite diagnosticar un hipotiroidismo subclínico

T4 libre y T3 normales y TSH elevada

25En caso de sospecha de síndrome poliglandular autoinmune con panhipopituitarismo, insufi ciencia suprarrenal y coma mixedematoso, ¿cómo debemos iniciar el tratamiento?

Primero administrar glucocorticoides y posteriormente levotiroxina, para evitar desencadenar una insufi ciencia suprarrenal

26 Tratamiento médico más utilizado para el hipotiroidismo Levotiroxina (L-T4)

27Parámetro más útil para control del tratamiento del hipotiroidismo primario

Determinación de TSH

28 Parámetro más útil para control del tratamiento del hipotiroidismo central

T4 libre

29 Objetivo del tratamiento del hipotiroidismo primario Normalización de niveles de TSH

Page 12: Preguntas de Endocrino

CTO Medicina • C/Núñez de Balboa 115 • 28006 Madrid • Tfno.: 91 782 43 30/33/34 • E-mail: [email protected] • www. grupocto.es 12

Test 3V Endocrinología y MetabolismoCTO MEDICINA

TEMA 3. ENFERMEDADES DE TIROIDES

30

Varón de 75 años que es traído a urgencias en coma. Presenta palidez, hinchazón de cara, pies y manos, hipotonía y arrefl exia, TA de 80/50 mmHg y temperatura rectal de 34º C, hemograma y bioquímica normales, ¿cuál debe ser la medida terapéutica más urgente?

Hormonas tiroideas y corticoides

31Tratamiento más adecuado ante la sospecha de un coma mixedematos

L-T4 intravenosa más hidrocortisona, para evitar que se desencadene una crisis suprarrenal

32Actitud más adecuada ante un paciente anciano, con antecedentes de infarto de miocardio hace 5 años, que presenta una TSH de 6 y una T4 libre normal

Vigilancia

33Mujer de 33 años de edad que en la analítica presenta unos niveles de T3 y T4 normales y una TSH de 11. Actitud más adecuada

Tratamiento sustitutivo con levotiroxina

34Causa más frecuente de hipertiroidismo en adulto en edades medias de la vida

Enfermedad de Graves

35 Causa más frecuente de hipertiroidismo en el anciano Bocio multinodular hiperfuncionante

36Anticuerpos que encontramos en el suero en la enfermedad de Graves

Anticuerpos IgG estimulantes de la tiroides (TSI)

37 Haplotipos relacionados con la enfermedad de Graves HLA DR3 y HLA B8

38 Histología típica de la enfermedad de GravesHipertrofi a e hiperplasia del parénquima tiroideo con infi ltración de linfocitos

39 Relaciona cada una de las siguientes patologías con su imagen gammagráfi ca característica:

- Gammagrafía hipercaptante de forma global: enfermedad de Graves, tumores productores de TSH y tumores productores de hCG- Captación irregular con múltiples nódulos en diversos estados funcionales: bocio multinodular tóxico- Nódulo único hipercaptante que suprime el resto de la glándula: adenoma tóxico- Captación disminuida: tiroiditis, fenómeno de Jod-Basedow, tirotoxicosis facticia, struma ovarii y metástais funcionales de carcinoma folicular

40Entidades que pueden cursar con clínica típica de hipertiroidismo y aumento de hCG

Mola hidatiforme y coriocarcinoma

41

Mujer de 45 años que es vista en urgencias por fi brilación auricular. Mide 158 cm, pesa 98 kg, y tiene una TA de 140/60 mmHg. No presenta adenopatías ni bocio, la T4 libre está alta y la TSH suprimida, la captación de yodo en la gammagrafía es baja y la tiroglobulina menor de 1 ng/ml (normal 1-30 ng/ml), diagnóstico más probable

Ingesta subrepticia de tiroxina

42

Varón de 47 años que ingresa en urgencias refi riendo palpitaciones, debilidad muscular, nerviosismo, diarrea y pérdida de peso. La semana pasada inició tratamiento con amiodarona. Los niveles de IL-6 se encuentran aumentados y la gammagrafía está abolida, ¿cuál es la sospecha diagnóstica más probable?

Tiroiditis por amiodarona tipo 2

Page 13: Preguntas de Endocrino

CTO Medicina • C/Núñez de Balboa 115 • 28006 Madrid • Tfno.: 91 782 43 30/33/34 • E-mail: [email protected] • www. grupocto.es 13

Test 3V Endocrinología y MetabolismoCTO MEDICINA

43 ¿Y su tratamiento más adecuado será? Glucocorticoides

44

Varón de 80 años de edad que presenta desde hace un mes pérdida de peso, debilidad muscular, depresión, lentitud y síntomas de insufi ciencia cardíaca sin otros antecedentes de interés, ¿cuál sería la actitud más adecuada?

Determinación de hormonas tiroideas y THS para descartar hipertiroidismo enmascarado

45Exoftalmos y placas de coloración rojiza o amarillenta en parte anterior de las piernas. Patología más probable

Enfermedad de Graves

46Utilidad de los glucocorticoides en el tratamiento del hipertiroidismo

Crisis tirotóxica y en la oftalmopatía de Graves severa

47 Método de screening inicial para diagnóstico de hipertiroidismo Determinación de TSH

48 Niveles hormonales en el diagnóstico de hipertiroidismo primario

T4 libre elevada y TSH suprimida

49 Niveles hormonales en el hipertiroidismo central y en la resistencia a hormonas tiroideas

T4 libre elevada y TSH elevada o normal

50 Prueba de utilidad para diferenciar el hipertiroidismo central de la resistencia a hormonas tiroideas: test de TRH

En el hipertiroidismo central hay poca respuesta de TSH a la TRH, mientras que sí hay respuesta en la resistencia a hormonas tiroideas

51 Tipos de tratamiento farmacológico del hipertiroidismoAntitiroideos, yoduro inorgánico, glucocorticoides y yodo radiactivo

52 Efecto secundario más grave del tratamiento con antitiroideos Agranulocitosis

53Mujer de 33 años que hace dos semanas inició un tratamiento con propiltiouracilo, y ahora mismo presenta fi ebre y dolor de garganta, ¿qué habría que sospechar?

Agranulocitosis por propiltiouracilo

54 ¿En qué consiste el efecto Wolff -Chaikoff ?En que el yoduro es capaz de inhibir su propio transporte en las células tiroideas e inhibir la organifi cación, impidiendo así la síntesis de yodotirosinas y la liberación de hormonas tiroideas

55 Efectos indeseables más importantes del radioyodo Persistencia del hipertiroidismo y desarrollo de hipotiroidismo

56Tratamiento que es necesario seguir antes y después de administrar una dosis ablativa de radioyodo

Antitiroideos

57 Contraindicaciones del radioyodoEmbarazo y lactancia, bocios grandes retroesternales, si existen dudas de malignidad, edad menor a 20 años, oftalmopatía de Graves severa y presencia de nódulo frío en la gammmagrafía

58Tratamiento de elección en la enfermedad de Graves en mujer de 30 años

Antitiroideos

59 Tratamiento de elección en varón de 50 años con enfermedad de Graves

Radioyodo

60Tratamiento de elección para un varón de 43 años con enfermedad de Graves, bocio compresivo y exoftalmos grave

Cirugía (tiroidectomía subtotal)

Page 14: Preguntas de Endocrino

CTO Medicina • C/Núñez de Balboa 115 • 28006 Madrid • Tfno.: 91 782 43 30/33/34 • E-mail: [email protected] • www. grupocto.es 14

Test 3V Endocrinología y MetabolismoCTO MEDICINA

61Actitud más correcta ante un mujer de 33 años, en su segundo embarazo, con enfermedad de Graves que no se controla con propiltiouracilo

Cirugía en el segundo trimestre de gestación

62Mujer de 33 años que tras un tratamiento prolongado con metimazol durante 18 meses no consigue una remisión completa de su enfermedad, ¿cuál es la actitud más correcta?

Utilización de yodo radiactivo

63 Tratamiento de elección en el bocio multinodular tóxico Radioyodo, y si es normofuncionante, seguimiento

64 Tratamiento de elección en el adenoma tóxicoRadioyodo o hemitiroidectomía quirúrgica con estudio histológico de la pieza

65¿Cuál es el betabloqueante de elección para el control de los síntomas en una crisis tirotóxica?

El propranolol

66 ¿Por qué? Porque inhibe la conversión periférica de T4 a T3

67Varón con adenoma tóxico, que tras recibir una dosis de I131 comienza con palpitaciones y fi brilación auricular, ¿cuál es el tratamiento de elección?

Antitiroideos, digital y betabloqueantes

68 Tratamiento de la crisis tirotóxica Antitiroideos, yodo, propranolol y dexametasona en dosis altas

69 Complicaciones derivadas de la tiroidectomíaDisfonía por daño del nervio laríngeo recurrente, sangrado masivo, hipoparatiroidismo e hipotiroidismo permanente

70Paciente que tras realizársele una tirodectomía subtotal por enfermedad de Graves presente una calcemia de 7 mg/dl, ¿qué es lo más probable?

Hipoparatiroidismo transitorio

71

Mujer de 42 años de edad que desde hace dos días presenta dolor, calor y enrojecimiento en la cara anterior del cuello, acompañado de fi ebre y leucocitosis, ¿cuál es el diagnóstico más probable?

Tiroiditis aguda bacteriana

72 Tratamiento más adecuado Antibióticos y drenaje quirúrgico si existe colección purulenta

73Clínica característica de la tiroiditis subaguda viral o de De Quervain

Dolor cervical anterior y aumento de la sensibilidad local, malestar general, fi ebre y aumento de la VSG

74 Gammagrafía típica de las tiroiditis Captación nula de yodo

75 Tratamiento de la tiroiditis viralSintomático, AAS, corticoides y propranolol para controlar síntomas de hipertiroidismo

76 Tratamiento contraindicado en las tiroiditis Antitiroideos

77Mujer de 50 años que presenta bocio no doloroso, captación nula en la gammagrafía y títulos bajos de antiperoxidasa, diagnóstico más probable

Tiroiditis linfocitaria silente o indolora

78Tipo de tiroiditis en que existe un mayor riesgo de desarrollar linfoma tiroideo

Tiroiditis linfocitaria crónica o de Hashimoto

Page 15: Preguntas de Endocrino

CTO Medicina • C/Núñez de Balboa 115 • 28006 Madrid • Tfno.: 91 782 43 30/33/34 • E-mail: [email protected] • www. grupocto.es 15

Test 3V Endocrinología y MetabolismoCTO MEDICINA

79

Mujer de 35 años con bocio indoloro, de consistencia elástica, con cifras de T4 libre normal y TSH ligeramente elevada y títulos elevados de anticuerpos antiperoxidasa. Diagnóstico más probable y tratamiento

Tiroiditis de Hashimoto y tiroxina

80

Varón de 51 años, con bocio inmóvil y de consistencia pétrea, sin afectación de ganglios linfáticos regionales, anticuerpos antitiroideos negativos y captación de yodo disminuida. Diagnóstico más probable

Tiroiditis de Riedel

81 Tratamiento de la tiroiditis de Riedel Cirugía si existen síntomas compresivos

82 Incidencia de carcinoma tiroideo en los nódulos tiroideos únicos

5%

83 ¿Y en los bocios multinodulares? 5%

84 Factores de riesgo de cáncer de tiroides ante un nódulo tiroideo

Antecedentes familiares de cáncer de tiroides o MEN 2, radiación de cabeza y cuello en la infancia, défi cit de yodo, nódulo de crecimiento rápido e indoloro, parálisis de cuerdas vocales o disfonía, sexo masculino, tamaño mayor de 4 cm, adenopatías palpables y fi jación a estructuras profundas

85 Datos radiológicos sugerentes de carcinomaNódulo frío hipocaptante en gammagrafía y nódulos mixtos o sólidos en ecografía

86¿Cuál es la variante histológica más frecuente en el linfoma tiroideo?

Linfoma histiocítico de células grandes

87 Tumor tiroideo más frecuente Carcinoma papilar

88 Tumor tiroideo con peor pronóstico Carcinoma anaplásico

89Tumores tiroideos no derivados del epitelio folicular de la tiroides

Carcinoma medular de tiroides y linfoma tioroideo

90 Relaciona las siguientes características con el tumor tiroideo con el que se asocian:

- Más frecuente en áreas ricas en yodo: carcinoma papilar- Diseminación linfática: carcinoma papilar- Diseminación vía hematógena: carcinoma folicular- Crecimiento rápido y ulceración de la piel: carcinoma anaplásico- Tiene un subtipo histológico, más invasor y menos radiosensible: carcinoma folicular- Tumor muy radiosensible: linfoma tiroideo- Presencia de tejido amiloide en la histología: carcinoma medular de tiroides- La calcitonina es el marcador tumoral para detectar enfermedad residual: carcinoma medular de tiroides-Cuerpos de psamoma o calcifi caciones en grano de arena: carcinoma papilar - Más frecuentemente relacionado con la radiación en la infancia: carcinoma papilar- Mayor riesgo de recurrencia: carcinoma papilar- Metástasis en pulmón, hueso y SNC: carcinoma folicular- Mutaciones en protooncogén RET: carcinoma medular

91Paciente que en seno de un MEN 2 presenta feocromocitoma y cáncer medular de tiroides, ¿qué patología hay que intervenir en primer lugar?

El feocromocitoma

Page 16: Preguntas de Endocrino

CTO Medicina • C/Núñez de Balboa 115 • 28006 Madrid • Tfno.: 91 782 43 30/33/34 • E-mail: [email protected] • www. grupocto.es 16

Test 3V Endocrinología y MetabolismoCTO MEDICINA

92 Tratamiento de elección en el cáncer medular de tiroides Tiroidectomía total con linfadenectomía central

93El método inicial más adecuado de diagnóstico para la mayoría de pacientes con un nódulo tiroideo único es

La punción aspiración con aguja fi na (PAAF)

94

Mujer de 45 años, diabética, que consulta por bulto en el cuello, asintomática, a la palpación se detecta un nódulo de aproximadamente 3 cm de diámetro; estudio bioquímico y hematológico normal, niveles de hormonas tiroideas normales, y en la gammagrafía el nódulo es hipocaptante, ¿qué habría que hacer a continuación?

Punción aspiración con aguja fi na del nódulo y estudio citológico

95 El resultado del estudio sugiere carcinoma papilar, ¿qué conducta es la más aconsejable?

Tiroidectomía total seguida de I131 y L-T4

96 Tratamiento de elección ante un carcinoma tiroideo Tiroidectomía total

97

Paciente de 60 años con nódulo tiroideo hipocaptante, con antecedentes familiares de cáncer de tiroides y disfonía de tres meses de evolución, al que se realiza una PAAF que es informada como dudosa, ¿Cuál es la actitud más aconsejable?

Extirpación amplia de la lesión, hemitiroidectomía con istmectomía y biopsia de la pieza

98Se demuestra la existencia de carcinoma papilar, ¿cuál debe ser la actitud a seguir?

Nueva cirugía para extraer restos tiroideos

99Tratamiento que deben seguir los pacientes operados de cáncer de tiroides

Levotiroxina en dosis supresoras que mantengan la TSH baja o indetectable

100Previamente a la administración de I131 para destruir tejido tiroideo residual tras una cirugía, ¿qué debemos hacer?

Suspender el tratamiento con hormona tiroidea para elevar los niveles plasmáticos de TSH

101Métodos de seguimiento tras una tiroidectomía por cáncer de tiroides

Rastreo corporal total con radioyodo y determinación de tiroglobulina

102 Marcador para detectar enfermedad residual en cáncer dife-renciado de tiroides

Tiroglobulina

103Un elevación de la tiroglobulina por encima de 10 ng/ml en pacientes que siguen tratamiento supresor, ¿de qué es sugestiva?

De la existencia de metástasis

104

Paciente de 49 años al que se le practicó una tiroidectomía por carcinoma folicular de tiroides hace un año, que en una de las revisiones periódicas encontramos pequeñas zonas pul-monares que captan radioyodo en el rastreo corporal total, y niveles de tiroglobulina dentro de la normalidad, ¿cuál será la actitud adecuada en este caso?

Determinar existencia de anticuerpos antitiroglobulina, ya que pueden falsear los niveles de tiroglobulina, y administrar una dosis ablativa de radioyodo para eliminar la enfermedad residual

TEMA 4. ENFERMEDADES DE LAS GLÁNDULAS SUPRARRENALES

1 Causa más frecuente de síndrome de Cushing Administración yatrógena de esteroides

2¿Cómo podemos diferenciar un síndrome de Cushing endógeno del síndrome de Cushing exógeno por administración de esteroides?

Porque en el Cushing exógeno existe clínica de Cushing pero con ACTH, cortisol libre en orina y cortisol plasmáticos suprimidos

3¿Cuál es la causa más frecuente de síndrome de Cushing endógeno?

El síndrome de Cushing hipofi sario por tumor hipofi sario, generalmente microadenoma

Page 17: Preguntas de Endocrino

CTO Medicina • C/Núñez de Balboa 115 • 28006 Madrid • Tfno.: 91 782 43 30/33/34 • E-mail: [email protected] • www. grupocto.es 17

Test 3V Endocrinología y MetabolismoCTO MEDICINA

4Cuando hablamos de enfermedad de Cushing nos estamos refi riendo a…

Síndrome de Cushing de origen hipofi sario

5 Otras causas de síndrome de Cushing endógeno menos frecuentes son

Síndrome de Cushing suprarrenal y síndrome de Cushing ectópico

6¿Cuáles son los tumores que más frecuentemente producen síndrome de Cushing?

Los tumores carcinoides (pulmón, timo, intestino, páncreas y ovario) y el carcinoma microcítico de pulmón

7 Mecanismo por el cuál algunas neoplasias producen síndrome de Cushing

Por producción autónoma de ACTH o CRH por las células tumorales

8 Características clínicas del síndrome de CushingObesidad central, HTA, intolerancia a la glucosa, cara de luna llena, estrías rojo-vinosas, acné, hirsutismo, trastornos menstruales, debilidad muscular, osteoporosis…

9 Manifestaciones clínicas del Cushing ectópico agresivoHiperglucemia, hipopotasemia y alcalosis metabólica, además de hiperpigmentación y miopatía proximal, pero sin el característico fenotipo cushingoide

10 Datos de laboratorio en el síndrome de Cushing de cualquier etiología

- Niveles plasmáticos y urinarios de cortisol y sus metabolitos aumentados (excepto en síndrome de Cushing yatrógeno, que están disminuidos)- Leucocitosis con neutrofi lia y eosinopenia- Hiperglucemia o diabetes franca- Alcalosis metabólica hipopotasémica con hipocloremia

11Pruebas de screening ante la sospecha clínica de un síndrome de Cushing

- Excreción de cortisol libre en orina durante 24 horas- Test de Nugent (supresión con 1 mg de dexametasona)

12Si las pruebas de screening son positivas, ¿qué prueba es la que debería realizarse a continuación?

Test de Liddle débil, administrando 2 mg/día durante dos días de dexametasona. Si el cortisol plasmático no suprime adecuadamente se hace el diagnóstico defi nitivo de síndrome de Cushing

13 Una vez realizado el diagnóstico de síndrome de Cushing, ¿cuál es el siguiente paso?

Medición de ACTH

14La presencia de ACTH < 10 (suprimida) en un paciente con síndrome de Cushing endógeno sugiere… y la siguiente prueba a realizar sería…

Tumor o hiperplasia suprarrenal…… TAC abdominal/suprarrenal

15Si la ACTH es detectable (>5-10 pg/ml) la siguiente prueba a realizar sería…

Test de 8 DXT de Liddle fuerte, con 2 mg/6 h durante dos días

16Si el cortisol plasmático se reduce por debajo del 90% de su valor basal, es sugestivo de…

Microadenoma hipofi sario secretor de ACTH

17Si no se reduce el cortisol plasmático por debajo del 90% de su valor basal es sugestivo de…

Macroadenoma hipofi sario productor de ACTH o tumores productores de ACTH ectópicaº

18Prueba de imagen de elección para la detección de tumores hipofi sarios

RM selar con gadolinio

19Para la detección de patología suprarrenal o de tumores ectópicos se emplea…

TAC corporal en cortes de 0,5 cm

Page 18: Preguntas de Endocrino

CTO Medicina • C/Núñez de Balboa 115 • 28006 Madrid • Tfno.: 91 782 43 30/33/34 • E-mail: [email protected] • www. grupocto.es 18

Test 3V Endocrinología y MetabolismoCTO MEDICINA

20Ante un síndrome de Cushing con niveles elevados de ACTH y donde no se aprecia una imagen tumoral clara en la RM, se debe realizar…

Un cateterismo bilateral de senos petrosos inferiores

21Si se observa un gradiente petroso-periférico de ACTH, con niveles de ACTH mayores en el seno petroso que en la vena periférica ¿dónde se encuentra el tumor productor de ACTH?

En la hipófi sis

22Existen algunos estados de pseudocushing que pueden plantear problemas diagnósticos. Cita algún ejemplo:

Obesidad severa, depresión, el alcoholismo crónico y enfermedades que causan gran estrés

23Tratamiento de elección en las neoplasias suprarrenales que producen síndrome de Cushing

Cirugía

24Tratamiento de elección de los tumores hipofi sarios productores de ACTH

Extirpación quirúrgica del tumor por vía transesfenoidal

25 Tratamiento del síndrome de Cushing de origen ectópico Extirpación quirúrgica del tumor

26Si estamos ante un síndrome de Cushing de origen ectópico cuyo tumor está oculto, ¿qué opciones de tratamiento existen?

- Suprarrenalectomía médica: ketoconazol, aminoglutetimida, mitotane o metopirona- Suprarrenalectomía quirúrgica bilateral (ocasionalmente) que precisa posteriormente tratamiento sustitutivo de por vida con mineralcorticoides y glucocorticoides

27 ¿A qué denominamos síndrome de Nelson?Al crecimiento de un tumor hipofi sario residual tras la suprarrenalectomía bilateral en pacientes con enfermedad de Cushing

28 Causas de insufi ciencia suprarrenal

- Enfermedad a nivel suprarrenal: enfermedad de Addison- Enfermedad hipotalámica o hipofi saria con défi cit de ACTH o CRH- Supresión prolongada del eje hipotálamo-hipófi so-adrenal por administración exógena de esteroides

29¿Cuál es la causa más común de insufi ciencia suprarrenal primaria?

La adrenalitis autoinmune

30Causa más frecuente de insufi ciencia suprarrenal en series antiguas

Tuberculosis

31 Causas de insufi ciencia suprarrenal típicas de pacientes con SIDA

Adrenalitas por Citomegalovirus, Mycobacterium avium intracellulare, o por criptococo, y sarcoma de Kaposi

32 Clínica sugestiva de insufi ciencia suprarrenalAstenia, debilidad, hipoglucemia, pérdida de peso, dolor abdominal, náuseas, vómitos…

33Diferencias entre la insufi ciencia suprarrenal primaria y secundaria

En la primaria existe aumento de la secreción de ACTH y sus péptidos, que dan lugar a hiperpigmentación mucocutánea, y existe también hipoaldosteronismo, con pérdida de sodio, hiperpotasemia, acidosis metabólica y avidez por la sal

34 Datos de laboratorio de la insufi ciencia suprarrenalDisminución de niveles de sodio, cloro y bicarbonato, y aumento del potasio en suero. Son frecuentes la hipoglucemia, anemia normocítica, linfocitosis y eosinofi lia

35 Causa más frecuente de una crisis suprarrenal aguda Suspensión brusca de un tratamiento corticoideo

Page 19: Preguntas de Endocrino

CTO Medicina • C/Núñez de Balboa 115 • 28006 Madrid • Tfno.: 91 782 43 30/33/34 • E-mail: [email protected] • www. grupocto.es 19

Test 3V Endocrinología y MetabolismoCTO MEDICINA

36 Clínica de la crisis adrenalFiebre elevada, deshidratación, náuseas, vómitos, hipotensión, shock…

37Paciente que ha estado recibiendo tratamiento con hidrocortisona hasta hace un mes, y que es ingresado en el hospital por una neumonía, ¿qué se debe hacer?

Aumentar dosis de esteroides de manera profi láctica para evitar una crisis suprarrenal

38 Ante la sospecha de una insufi ciencia suprarrenal, ¿qué es lo primero que se debe realizar?

Medición de niveles basales de cortisol y aldosterona en sangre y ACTH

39 La concentración de ACTH estará elevada en la… y disminui-da o normal en la…

Insufi ciencia suprarrenal primaria….. insufi ciencia suprarrenal secundaria

40¿Cuál es la prueba de referencia para el diagnóstico de insufi ciencia suprarrenal primaria?

Prueba de estimulación con 250 ug de ACTH. Es normal si el cortisol se eleva por encima de 18 ug/dl a los 30 ó 60 minutos; si no se eleva, se hace el diagnóstico de insufi ciencia suprarrenal primaria

41¿Cuál es la prueba diagnóstica más fi able para el diagnóstico de insufi ciencia suprarrenal secundaria?

La hipoglucemia insulínica (prueba de función hipofi saria)

42Si en la prueba de estimulación con ACTH el cortisol no responde pero sí lo hace la aldosterona, ¿cuál será el diagnóstico más probable?

Insufi ciencia suprarrenal secundaria

43Causas más frecuentes de destrucción hemorrágica de las glándulas suprarrenales?

En los niños: septiciemia por pseudomonas y meningococemia; en adultos: tratamiento anticoagulante

44 Tratamiento de la enfermedad de AddisonGlucocorticoides de vida media corta o media (cortisona, hidrocortisona o prednisona) y mineralcorticoides (fl udrocortisona)

45Paciente con enfermedad de Addison al que se diagnostica un carcinoma colorrectal y va a ser sometido a colectomía, ¿qué hay que tener en cuenta antes de la cirugía?

Que es necesario elevar la administración de glucocorticoides hasta 10 veces la dosis habitual

46 Tratamiento de la insufi ciencia suprarrenal secundariaTratamiento sustitutivo con glucocorticoides y otras hormonas si hay défi cits hormonales asociados. Generalmente no se requiere tratamiento con mineralcorticoides

47 Tratamiento en el caso de una crisis suprarrenalBolo de hidrocortisona inicialmente, manteniendo una perfusión continua posteriormente, y reposición de sodio y agua mediante infusión de solución salina fi siológica

48 La hidrocortisona en dosis elevadas tiene también efecto… Mineralcorticoide

49 Causa más frecuente de hiperaldosteronismo primario Adenoma suprarrenal productor de aldosterona (síndrome de Conn)

50 Causas de hiperaldosteronismo secundarioPérdida de sodio, depleción de volumen, disminución de fl ujo sanguíneo renal o tumores productores de renina

51 Diagnóstico diferencial de los hiperaldosteronismos

- En el hiperaldosteronismo primario está aumentada la aldosterona y disminuida la ARP (actividad de renina plasmática).- En el hiperaldosteronismo secundario está aumentada la aldosterona y la ARP- Otras causas de hipermineralcorticismo cursan con aldosterona y ARP disminuidas (síndrome de Liddle, tumores productores de DOCA…)

Page 20: Preguntas de Endocrino

CTO Medicina • C/Núñez de Balboa 115 • 28006 Madrid • Tfno.: 91 782 43 30/33/34 • E-mail: [email protected] • www. grupocto.es 20

Test 3V Endocrinología y MetabolismoCTO MEDICINA

52 Clínica fundamental del hiperaldosteronismo Hipertensión arterial

53Datos de laboratorio fundamentales para el diagnóstico de hiperaldosteronismo

- Hipopotasemia, eliminación exagerada de potasio en orina- Hipernatremia- Alcalosis metabólica

54 Indicaciones para realizar un screening de hiperaldosteronismo

- Pacientes con HTA e hipokalemia severa espontánea o inducida por diuréticos- Paciente con HTA sin tratamiento, con potasio bajo o en el límite bajo- Incidentalomas suprarrenales- HTA refractaria a tratamiento médico- Niños o jóvenes con HTA

55¿Cuál es el primer paso que debe realizarse para iniciar el diagnóstico de un hiperaldosteronismo?

Suspender el tratamiento antihipertensivo previo entre 2-4 semanas y replecionar al paciente de sodio y potasio antes de la evaluación

56 La primera medición que debe hacerse para el diagnóstico de un hiperaldosteronismo es…

Relación aldosterona plasmática (ng/dl) / ARP (ng/ml/h). Si es mayor de 30-50 con niveles de aldosterona en plasma > 20 ng/dl habrá que proseguir el estudio

57Si tras la infusión de suero salino fi siológico o tras la administración de un IECA, la aldosterona sigue en niveles superiores superiores a 20 ng/dl, ¿qué debemos sospechar?

Un hiperaldosteronismo primario

58¿Cómo se realiza el diagnóstico diferencial entre un hiperaldosteronismo primario y uno secundario?

Midiendo los niveles de renina

59 Los niveles de renina estarán elevados en un hiperaldosteronismo… y estarán suprimidos en uno…

Secundario….primario

60Tras el diagnóstico bioquímico de hiperaldosteronismo, ¿cuál es la siguiente prueba a realizar?

Pruebas de imagen (TAC abdominal) o pruebas invasivas (menos utilizadas): cateterismo de venas suprarrenales

61 Tratamiento fundamental del hiperaldosteronismo

- Si existe adenoma: extirpación quirúrgica- Si está contraindicada la cirugía: restricción de sodio y administración de espironolactona o eplerenona- En el hiperaldosteronismo por hiperplasia bilateral: tratamiento farmacológico: espironolactona, triamtirene, amiloride

62 Causa más frecuente de hipoaldosteronismo aislado Défi cit de producción de renina

63 Clínica del hipoaldosteronismo Hiperpotasemia y acidosis metabólica

64El hipoaldosteronismo hiporreninémico es típico de pacientes con…

Pacientes diabéticos con nefropatía

65 Pruebas diagnósticas del hipoaldosteronismoTest postural y test de deambulación más furosemida: no aumentan ni la renina ni la aldosterona

66 Tratamiento del hipoaldosteronismo- Si paciente hipertenso: furosemida- Si paciente normo o hipotenso: fl udrocortisona

67Lo primero que debe realizarse ante la existencia de una masa suprarrenal es

- Determinar si es funcionante o no mediante determinaciones basales hormonales y pruebas funcionales- Descartar malignidad mediante pruebas de imagen (TAC) y PAAF

Page 21: Preguntas de Endocrino

CTO Medicina • C/Núñez de Balboa 115 • 28006 Madrid • Tfno.: 91 782 43 30/33/34 • E-mail: [email protected] • www. grupocto.es 21

Test 3V Endocrinología y MetabolismoCTO MEDICINA

68No se debe realizar una PAAF de una masa suprarrenal ante la sospecha de…

Feocromocitoma

69 La PAAF tiene mayor efi cacia en el diagnóstico de… Metástasis suprarrenales

70 Indicaciones de cirugía de una masa suprarrenal- Tumores funcionantes- Masas > 4 cm

71Paciente que en la realización de TAC se descubre adenoma no funcionante menor de 4 cm de diámetro, ¿cuál es la actitud más adecuada?

Evaluación con TAC cada 6 -12 meses

72 Causa ovárica más frecuente de hiperandrogenismo Síndrome de ovario poliquístico

73 Clínica del síndrome de ovario poliquísticoAcné, hirsutismo (no virilización), con elevación de testosterona, androstendiona, oligomenorrea y anovulación

74El inicio súbito de hirsutismo y virilización progresivas sugiere…

Una neoplasia suprarrenal u ovárica

75 Diferencias bioquímicas entre una neoplasia suprarrenal y una ovárica

- En los carcinomas suprarrenales virilizantes hay niveles elevados de DHEA-S y eliminación aumentada de 17 cetosteroides- En los carcinomas ováricos la DHEA-S y los 17-cetosteroides están normales y hay aumento de testosterona

76 Tumor ovárico virilizante más frecuente Arrenoblastoma

77 ¿Dónde se origina el feocromocitoma?En la médula suprarrenal y ganglios simpáticos (paragangliomas)

78 Forma de presentación más frecuente del feocromocitoma Como lesión única y unilateral

79Los feocromocitomas suprarrenales secretan… mientras que cuando son extrasuprarrenales secretan sobre todo…

Adrenalina y noradrenalina…… noradrenalina

80 Manifestación clínica más frecuente del feocromocitomaHTA (de forma mantenida o en paroxismos hipertensivos) y casi siempre resistente al tratamiento convencional

81 Causas desencadenantes de una crisis hipertensiva en paciente portador de feocromocitoma

Actividades que desplazan el contenido abdominal, el estrés, el ejercicio físico, cambios posturales, maniobras de Valsalva, alimentos como el queso, alcohol…

82

Varón de 43 años que es llevado a Urgencias por un cuadro de palpitaciones, sudoración y palidez facial, sin alteraciones neurológicas, que el paciente asocia a la micción. TA 170/110 mmHg, glucemia de 140 mg/dl, ¿qué hay que descartar?

Un feocromocitoma vesical

83 Paciente con manchas café con leche, anomalías vertebrales e HTA, ¿qué habrá que descartar?

Enfermedad de von Recklinghausen (neurofi bromatosis tipo I)

84Ante la sospecha de feocromocitoma,.¿cuáles son las pruebas diagnósticas fundamentales?

Medición de catecolaminas libres y sus metabolitos (ácido vanidilmandélico y metanefrinas) en orina de 24 horas y pruebas de imagen para localizar el tumor (TAC y RM abdominal o gammagrafía con metayodobenzilguanidina para detectar feocromocitoma extrasuprarrenal)

Page 22: Preguntas de Endocrino

CTO Medicina • C/Núñez de Balboa 115 • 28006 Madrid • Tfno.: 91 782 43 30/33/34 • E-mail: [email protected] • www. grupocto.es 22

Test 3V Endocrinología y MetabolismoCTO MEDICINA

85Tratamiento preoperatorio fundamental ante la cirugía de un feocromocitoma

Inducción de bloqueo alfaadrenérgico estable y prolongado con fenoxibenzamina 10-14 días antes de la intervención

86Si existen crisis hipertensivas en el tiempo preoperatorio antes de conseguir bloqueo alfa completo, o durante la cirugía, estas se tratan mediante…

Fentolamina iv

87Los betabloqueantes están contraindicados en el feocromocitoma hasta que no se logra…

El bloqueo alfa completo

88En el tratamiento de las arritmias inducidas durante la cirugía del Feocromocitoma por liberación de catecolaminas o durante la inducción anestésica, se utilizan…

Bloqueantes beta como el propranolol

89La expansión de volumen mediante la administración de sal y suero salino antes de la cirugía del Feocromocitoma ayuda a evitar…

La hipotensión postquirúrgica

TEMA 5. DIABETES MELLITUS

1 Enfermedad endocrina más frecuente Diabetes mellitus (DM)

2 Forma más frecuente de diabetes mellitus Tipo 2

3 ¿En qué tipo de diabetes mellitus son más frecuentes las complicaciones crónicas en el momento del diagnóstico?

En el tipo 2

4Glucemia plasmática a las dos horas de la sobrecarga oral de glucosa con 75 g, entre 140 y 199 mg/dl, es compatible con…

Intolerancia a los hidratos de carbono

5 Criterio de glucosa alterada en ayunas Glucemia plasmática en ayunas entre 100 y 125 mg/dl

6

Si un paciente no diagnosticado previamente de diabetes mellitus es ingresado en urgencias con un coma hiperosmolar, ¿cuál es la prueba que se debe realizar para diagnosticar al paciente?

Ninguna, directamente se diagnostica de diabetes mellitus

7Test de screening en la población general para el diagnóstico de DM

Glucosa plasmática en ayunas

8 En pacientes que presentan la glucosa alterada en ayunas de forma repetida, ¿qué prueba se debe realizar?

Una sobrecarga oral de glucosa (SOG)

9 Criterios diagnósticos de la DM- Glucemia > o igual 200 con clínica cardinal- Glucemia > o igual 126 en ayunas (dos determinaciones)- Glucemia > o igual 200 a las 2 horas de una SOG

10 ¿A qué pacientes se debe realizar cribado de diabetes mellitus tipo 2 cada 3 años, estando asintomáticos?

- Pacientes mayores de 45 años- Pacientes menores de 45 años con factores de riesgo: sobrepeso, historia familiar de DM, sedentarismo…

11 Etiopatogenia fundamental de la DM tipo 1Défi cit de la secreción de insulina por destrucción inmunológica del páncreas

12 Prueba diagnóstica del défi cit de secreción de insulinaNiveles de péptido C tras el estímulo con glucagón. Serán muy bajos en la DM tipo 1

Page 23: Preguntas de Endocrino

CTO Medicina • C/Núñez de Balboa 115 • 28006 Madrid • Tfno.: 91 782 43 30/33/34 • E-mail: [email protected] • www. grupocto.es 23

Test 3V Endocrinología y MetabolismoCTO MEDICINA

13 Estos pacientes precisan tratamiento desde el momento del diagnóstico con…

Insulina

14La aparición de manifestaciones clínicas en la DM tipo 1 se produce cuando…

Se han destruido más del 90% de células pancreáticas

15¿En cuál de los dos tipos de diabetes mellitus tienen más importancia los antecedentes familiares?

En la DM tipo 2

16Haplotipos del HLA a los que se ha asociado la aparición de DM tipo 1

HLA DR3 y DR4

17 ¿Qué factor ambiental podría estar relacionado con el inicio de la destrucción pancreática que da lugar a la DM tipo 1?

Una infección vírica

18 Anticuerpos que se emplean como marcadores de la DM tipo 1

- Anticuerpos contra las células de los islotes (ICA)- Anticuerpos frente a la descarboxilasa del ácido glutámico (GAD)- Anticuerpos antiinsulina (AAI)- Anticuerpos frente a una fosfatasa de proteina-kinasa (IA-2)

19 ¿Cuáles de ellos son los que tienen mayor rentabilidad diagnóstica y se usan actualmente?

Los anticuerpos frente a la descarboxilasa del ácido glutámico

20 Etiopatogenia fundamental de la DM tipo 2Se debe a una resistencia a la acción de la insulina en tejidos periféricos y a un défi cit en la secreción de insulina por el páncreas (défi cit relativo)

21 ¿Cómo es la masa de células beta pancreáticas en este tipo de DM?

Normal

22 Tipo de herencia en la DM tipo 2 Poligénica, no muy bien conocida

23 Factores ambientales con los que se relaciona la DM tipo 2Obesidad, envejecimiento, inactividad física y dietas hipercalóricas

24 Tratamiento inicial de la DM tipo 2 Dieta y antidiabéticos orales si no se controla con la dieta

25 Etiopatogenia de la DM tipo MODY Mutación de genes que intervienen en la secreción de insulina

26 La DM tipo MODY se caracteriza porInicio en edad precoz, generalmente antes de los 25 años, con hiperglucemia leve y sin cetosis

27 Tipos más frecuentes de DM tipo MODYEl tipo 2 (mutaciones en el gen de la glucokinasa) y tipo 3 (mutaciones en el gen del factor nuclear hepático 1-alfa)

28 Cita cuatro fármacos que puedan producir DMGlucocorticoides, tiacidas, fenitoína, anticoagulantes orales y pentamidina

29 ¿Cómo se defi ne la resistencia insulínica severa?Como la necesidad de más de 200 UI al día de insulina o de más de 1,5 UI/kg/día para controlar la glucemia

30 Uno de los síndromes de resistencia insulínica se trata con corticoides e inmunosupresores, ¿cuál?

El tipo B

31 ¿Cuál de los síndromes de resistencia insulínica es frecuente en mujeres con hiperandrogenismo y acantosis nigricans?

El tipo A

Page 24: Preguntas de Endocrino

CTO Medicina • C/Núñez de Balboa 115 • 28006 Madrid • Tfno.: 91 782 43 30/33/34 • E-mail: [email protected] • www. grupocto.es 24

Test 3V Endocrinología y MetabolismoCTO MEDICINA

32 Clínica fundamental de la DM tipo 1Personas jóvenes, con poliuria, polifagia, polidipsia, astenia y pérdida de peso de comienzo brusco

33 ¿Cuál puede ser una forma de presentación típica en niños con DM?

Enuresis secundaria

34¿Qué caracteriza al periodo denominado “luna de miel” cuando se comienza el tratamiento con insulina?

Periodo de remisión parcial en que las necesidades de insulina son bajas y el control glucémico fácil de conseguir

35 Clínica fundamental de la DM tipo 2

Personas de edad intermedia o avanzada, se presenta de forma insidiosa y la mayoría de las veces se diagnostica como un ha-llazgo casual de hiperglucemia en pacientes asintomáticos. A veces se presenta como coma hiperosmolar

36 ¿Qué tipo de tratamiento comenzarías a utilizar inicialmente? Dieta + ejercicio físico + metformina

37

En un paciente con DM tipo 2 desde hace 7 años, en tratamiento con antidiabéticos orales y con cifras elevadas de glucosa en las glucemias capilares, se debería plantear iniciar tratamiento con

Insulina

38 Método de estimación de la reserva pancreática de insulinaRespuesta de péptido C a los 6 minutos del estímulo con glucagón

39 Principales complicaciones agudas de la diabetes mellitusLa hipoglucemia, la cetoacidosis diabética y la descompensación hiperosmolar

40 La cetoacidosis diabética es más frecuente en… La DM tipo 1

41La descompensación hiperosmolar es una complicación característica de…

La DM tipo 2

42 Factores desencadenantes de la cetoacidosis diabéticaAbandono del tratamiento con insulina, transgresiones dietéticas, infecciones, traumatismos, cirugía…

43 Clínica y datos de laboratorio típicos de la cetoacidosis

Náuseas, vómitos y dolor abdominal, y si no se trata precozmente, obnubilación y coma. Glucemia > 250 mg/dl, acidosis metabólica (pH < 7,30), aumento de cuerpos cetónicos en plasma, cuerpos cetónicos positivos en orina y bicarbonato plasmático disminuido, potasio disminuido

44Estado en que puede aparecer una hiponatremia con osmolaridad plasmática elevada

Hiperglucemia

45 ¿Qué se debe sospechar ante la presencia de fi ebre en una cetoacidosis diabética?

Una infección

46 Ante la sospecha de una cetoacidosis diabética, ¿cuáles son las determinaciones que hay que realizar en primer lugar?

Glucemia y cetonuria

47 Segunda causa más frecuente de cetoacidosis después de la DM

La cetoacidosis alcohólica

48 Tratamiento fundamental de la cetoacidosis diabéticaInsulina rápida o regular por vía iv en perfusión continua, e hidratación del paciente con suero salino y, posteriormente con suero glucosado al 5% cuando la glucemia es < de 250 mg/dl

Page 25: Preguntas de Endocrino

CTO Medicina • C/Núñez de Balboa 115 • 28006 Madrid • Tfno.: 91 782 43 30/33/34 • E-mail: [email protected] • www. grupocto.es 25

Test 3V Endocrinología y MetabolismoCTO MEDICINA

49 ¿Cuándo se puede retirar la insulina iv?Cuando se corrija la acidosis y después de 2 horas del inicio del tratamiento con insulina subcutánea

50 ¿Cuándo son necesarios los suplementos de potasio?Si la cifra de potasio es normal o baja, desde el inicio del tratamiento. Si el potasio es > 5,5-6 mEq/l esperar 3 ó 4 horas tras comenzar tratamiento

51 ¿Cuándo es necesaria la administración de bicarbonato? Si el pH es < de 7,00

52 Si tras el tratamiento no se obtiene respuesta en 4 ó 6 horas, ¿qué se debería sospechar?

Resistencia a la insulina

53 ¿Cómo se realiza el seguimiento de la cetoacidosis diabética?Mediante determinaciones seriadas de glucemia, pH, bicarbonato, anión gap e iones

54 Principales causas de muerte en la cetoacidosis diabética Infarto de miocardio y neumonía

55 En niños una causa frecuente de muerte es… El edema cerebral

56 Factores desencadenantes de un coma hiperosmolarGeneralmente ancianos diabéticos con un cuadro infeccioso, como neumonía o sepsis de origen urológico

57 Clínica de la descompensación hiperosmolarDeshidratación por diuresis osmótica, manifestaciones neurológicas y microtrombosis por el aumento de la viscosidad plasmática

58Datos de laboratorio más característicos de la descompensación hiperosmolar

Hiperglucemia > de 600 mg/dl, osmolaridad plasmática elevada, acidosis metabólica leve, niveles de bicarbonato alrededor de 20 mEq/l, generalmente sin aumento de cuerpos cetónicos

59Tanto en la cetoacidosis como en la descompensación hiperosmolar existe acidosis metabólica, ¿a qué es debido en cada uno de los casos?

- En la cetoacidosis, al aumento de cuerpos cetónicos en plasma, sobre todo acetoacetato e hidroxibutirato- En la descompensación hiperosmolar, debido al aumento del ácido láctico por mala perfusión tisular

60 Tratamiento fundamental de la descompensación hiperosmolarLa hidratación, inicialmente con suero salino y luego con suero glucosado al 5%

61 Otros tratamientos necesarios en la descompensación hiperosmolar

- Insulina iv (a dosis menores que en la cetoacidosis)- Potasio (iniciar su administración más precozmente que en la cetoacidosis), excepto si está elevado- Bicarbonato (sólo si existe acidosis láctica con pH< 7.20)- Antibióticos si se sospecha infección

62 Factores desencadenantes de hipoglucemiaOmisión o retraso de una comida, exceso de insulina o hipoglucemiantes orales, el ejercicio intenso…

63¿Con qué tipo de tratamiento es más frecuente que aparezcan hipoglucemias?

Con el tratamiento insulínico intensivo

64 Síntomas fundamentales que nos hacen sospechar una hipoglucemia

- Síntomas adrenérgicos: sudoración, nerviosismo, temblor, palidez, palpitaciones…- Síntomas neuroglucopénicos: cefalea, disminución de capacidad de concentración, trastornos de conducta y lenguaje, visión borrosa, convulsiones…

Page 26: Preguntas de Endocrino

CTO Medicina • C/Núñez de Balboa 115 • 28006 Madrid • Tfno.: 91 782 43 30/33/34 • E-mail: [email protected] • www. grupocto.es 26

Test 3V Endocrinología y MetabolismoCTO MEDICINA

65 Tratamiento de las hipoglucemias

- Si paciente consciente: administrar hidratos de carbono de absorción rápida vía oral- Si paciente inconsciente: glucagón por vía subcutánea o intramuscular y suero salino glucosado

66¿Qué tipo de tratamiento produce hipoglucemias muy prolongadas que requieren vigilancia hospitalaria durante 48 horas?

Las sulfonilureas

67 Principales complicaciones crónicas de la DM

- Microvasculares: retinopatía, neuropatía y nefropatía, gastroparesia, vejiga neurógena…- Macrovasculares: aterosclerosis acelerada, cardiopatía isquémica, enfermedad cerebrovascular y enfermedad arterial periférica

68 Los IAM silentes son más frecuentes en pacientes con Diabetes mellitus

69 ¿Cuál es el tratamiento del IAM con el que se han obtenido mejores resultados en pacientes con DM?

Bypass aortocoronario

70 Tratamiento de las úlceras diabéticasReposo, desbridamiento quirúrgico, curas locales y tratamiento antibiótico de amplio espectro

71 Infecciones típicas de pacientes diabéticos

- Otitis externa maligna por Pseudomonas aeruginosa- Mucormicosis rinocerebral- Colecistitis enfi sematosa- Pielonefritis enfi sematosa

72¿Qué es lo fundamental para prevenir el desarrollo de complicaciones crónicas de la DM y evitar su progresión?

Un estricto control metabólico de la glucemia

73 Los niveles de HbA1c se han relacionado con… El riesgo de complicaciones microangiopáticas

74 Además del control glucémico es necesario también un estricto control de…

Factores de riesgo cardiovascular asociados: obesidad, hipertensión arterial y dislipemia

75 Pilares del tratamiento del diabéticoDieta, ejercicio, fármacos, hipoglucemiantes orales e insulina, y el autocontrol

76 ¿Cómo debe ser la dieta en el diabético?

- Hipocalórica si es obeso- Hipercalórica si está desnutrido- Normocalórica si está en normopeso- Aumentar el consumo diario de fi bra soluble

77¿En qué tipo de diabetes es fundamental seguir adecuadamente la composición de la dieta?

En el tipo 2

78 Benefi cios del ejercicio en paciente diabéticoReduce necesidades de insulina, ayuda a mantener normopeso, reduce riesgo cardiovascular

79 Pautas de tratamiento insulínico convencionalUna o dos inyecciones de insulina de acción intermedia o prolongada (NPH, glargina o detemir) 2/3 de la dosis antes del desayuno y 1/3 antes de la cena

Page 27: Preguntas de Endocrino

CTO Medicina • C/Núñez de Balboa 115 • 28006 Madrid • Tfno.: 91 782 43 30/33/34 • E-mail: [email protected] • www. grupocto.es 27

Test 3V Endocrinología y MetabolismoCTO MEDICINA

80 Pautas de tratamiento insulínico intensivo

- Administración de insulina de acción corta (regular, lispro o aspártica) antes de cada comida para controlar el pico de glucemia postprandial junto con insulina de efecto prolongado o de acción intermedia en una o varias dosis al día- Infusión subcutánea continua de insulina de acción corta mediante una bomba situada en el tejido subcutáneo de la pared abdominal, que libera insulina a un ritmo basal a lo largo del día y se programa un pico de insulina antes de cada comida

81 Contraindicaciones del tratamiento insulínico intensivo

- Menores de 8 años- Diabéticos con neuropatía autonómica severa- Pacientes con trastornos mentales graves- Ancianos- Cardiópatas o pacientes con antecedentes de accidentes cerebrovasculares

82 Principales efectos secundarios de la insulina

- Hipoglucemia- Alergia a la insulina- Lipodistrofi a en el sitio de inyección- Insulinoresistencia- Edema insulínico- Presbicia insulínica

83 ¿A qué se denomina fenómeno de Somogyi?Hiperglucemia a primeras horas de la mañana por aumento de hormonas contrarreguladoras en respuesta a una hipoglucemia nocturna

84 Su tratamiento se basa en… Reducir dosis de insulina por la noche

85 ¿A qué se denomina fenómeno del alba?Hiperglucemia matutina debido a la secreción nocturna de GH o al ritmo circadiano del cortisol

86 Su tratamiento se basa en…Aumentar la dosis de insulina por la noche para mantener la normoglucemia

87 ¿Cómo se pueden distinguir ambos fenómenos?Mediante medición de glucemia plasmática a las 3 de la mañana. Si es baja, es un fenómeno de Somogyi; si es normal, un fenómeno del alba

88

Une cada hipoglucemiante oral con su mecanismo de acción:- Sulfonilureas- Meglitinidas- Biguanidas- Tiazolidinedionas- Inhibidores de las alfaglucosidasas

- Estimulan liberación de insulina por células beta pancreáticas- Regulan la secreción de insulina- Disminuyen resistencia insulínica a nivel hepático, potencian acción periférica de insulina y reducen absorción intestinal de glucosa- Disminuyen la resistencia insulínica a nivel muscular y del tejido graso- Retrasan absorción de hidratos de carbono, disminuyendo el pico glucémico postprandial

89 Efectos secundarios de las sulfonilureas Hipoglucemias severas y duraderas

90 El uso de sulfonilureas está indicado solamente en… DM tipo 2

91 Contraindicaciones de sulfonilureasEmbarazo, alergia a sulfamidas, hepatopatía avanzada o insufi ciencia renal

92 Principal indicación de las meglitinidas Pacientes con predominio de hiperglucemia postprandial

93 Contraindicaciones de meglitinidas Embarazo y hepatopatía severa

Page 28: Preguntas de Endocrino

CTO Medicina • C/Núñez de Balboa 115 • 28006 Madrid • Tfno.: 91 782 43 30/33/34 • E-mail: [email protected] • www. grupocto.es 28

Test 3V Endocrinología y MetabolismoCTO MEDICINA

94 La nateglinida en nuestro país se usa sobre todo asociada a… Metformina

95 Biguanida más usada en España Metformina

96 Efecto adverso más severo de la metformina Acidosis láctica

97 Efecto secundario más frecuente de la metformina Molestias gastrointestinales, náuseas y diarrea

98 Contraindicaciones de la metforminaSituaciones que favorezcan la acidosis láctica o deterioro de la función renal: alcoholismo, insufi ciencia cardiaca o respiratoria, hepatopatía, nefropatía, y en el embarazo

99 Efecto secundario fundamental de las glitazonas Hepatotoxicidad severa

100 Contraindicaciones de las glitazonas Insufi ciencia cardíaca, insufi ciencia hepática y embarazo

101 Efectos adversos de los inhibidores de las alfaglucosidasas Molestias gastrointestinales y fl atulencia

102 Contraindicaciones de los inhibidores de las alfaglucosidasas Menores de 18 años y embarazadas

103Paciente diabético en tratamiento con insulina y acarbosa, con hipoglucemia de 35 mg/dl, ¿qué debemos administrar para corregir la hipoglucemia?

Glucosa

104 Situaciones en que debe realizarse tratamiento insulínico de entrada en DM tipo 2

Sujetos delgados, que han sufrido pérdida de peso intensa, nefropatía o hepatopatía, y hospitalizados por enfermedad aguda

105¿Cuál es la mejor arma para la modifi cación de la pauta de tratamiento insulínico o con hipoglucemiantes orales?

El autocontrol domiciliario del paciente con glucemias capilares

106Mejor parámetro para evaluar el control glucémico a largo plazo

Hemoglobina glicada o hemoglobina A1c

107 Objetivos del control metabólico en un paciente diabético

- HbA1c < 7%- Glucemia preprandrial 70-130 mg/dl- Glucemia postprandrial < 180 mg/dl- TA < 130/80 mmHg- IMC < 25 kg/m2- LDL- col < 100 mg/dl- HDL-col > 40 mg/dl en varones y >50 mg/dl en mujer- HDL> 50 mg/dl en mujeres premenopáusicas- Triglicéridos < 150 mg/dl- Abandono del hábito tabáquico

TEMA 6. HIPOGLUCEMIAS

1 Tríada de Whipple característica de la hipoglucemia

- Existencia de síntomas de hipoglucemia (adrenérgicos y neuroglucopénicos)- Cifras bajas de glucosa- Mejoría de los síntomas tras administración de glucosa

2En las hipoglucemias de ayuno predominan los síntomas… y en las hipoglucemias postprandiales predominan síntomas…

Neuroglucopénicos ….. adrenérgicos

3 Causas de hipoglucemia de ayunoInsulinoma, défi cit hormonales (insufi ciencia suprarrenal, défi cit de GH), tumores extrapancreáticos (fi brosarcoma), hepatopatía severa, insufi ciencia renal severa…

Page 29: Preguntas de Endocrino

CTO Medicina • C/Núñez de Balboa 115 • 28006 Madrid • Tfno.: 91 782 43 30/33/34 • E-mail: [email protected] • www. grupocto.es 29

Test 3V Endocrinología y MetabolismoCTO MEDICINA

4 Causas de hipoglucemia reactiva o postprandialGastrectomía, intolerancia a los hidratos de carbono o DM, hipoglucemia reactiva verdadera

5 La hipoglucemia reactiva verdadera es típica de … y se debe a…

Mujeres jóvenes ….. liberación aumentada o retardada de insulina en respuesta a la ingesta

6 Tratamiento de la hipoglucemiaGlucosa por vía oral si el paciente está consciente. Si está inconsciente, glucosa por vía intravenosa

7Se debe descartar la existencia de un insulinoma ante la existencia de…

Hipoglucemia (casi siempre de ayuno), aumento de peso, y niveles normales o altos de insulina plasmática

8 Prueba diagnóstica para descartar la existencia de insulinomaPrueba de ayuno prolongado durante 72 horas y determinación de glucosa, insulina y péptido C

9 Se establece el diagnóstico de insulinoma, ¿ante qué resultados?

Hipersecreción exagerada de insulina (cociente insulina/glucosa>0,3) en presencia de hipoglucemia, y aumento del péptido C

10Si aparece una cifra de insulina alta y unos niveles de péptido C bajos, ¿qué deberíamos sospechar?

Hipoglucemia por insulina exógena

11Si aparecen en el estudio cifras de insulina y de péptido C altas, ¿qué hay que descartar? …¿y cómo?...

Tratamiento del paciente con sulfonilureas…… mediante la determinación de niveles de sulfonilureas en plasma y orina

12 El método más efi caz para la visualización de un insulinoma es…

TAC abdominal de alta resolución

13El método más efi caz para el diagnóstico de localización del insulinoma es….

La ecografía pancreática intraoperatoria

14 Ante un tumor (insulinoma) localizado, el tratamiento más adecuado es…

Enucleación del tumor

15 Ante un insulinoma no localizado se debe realizar…Pancreatectomía distal gradual dejando al menos un 20-30% de páncreas

TEMA 7. NUTRICIÓN, DISLIPEMIA Y OBESIDAD

1 ¿Cuál es el componente mayoritario de la dieta? Los carbohidratos en un 55%

2¿Cuáles son los carbohidratos más benefi ciosos y qué deberían aportarse con la dieta?

Los de absorción lenta, como los almidones, y evitar en exceso los de absorción rápida

3 Las grasas más benefi ciosas son… que proceden de… Las monoinsaturadas …… el aceite de oliva y los vegetales

4 Las grasas saturadas proceden de… Grasas animales

5 Las grasas poliinsaturadas proceden de… Grasas vegetales y pescado azul

6 ¿De qué depende el valor nutricional de una proteína? De la presencia de aminoácidos esenciales en su composición

7 ¿De qué depende el valor biológico de una proteína?De lo equilibrada que esté en cuanto a su composición de aminoácidos y a su digestibilidad

8 ¿Cuál es la proteína que presenta mayor valor nutricional?La ovoalbúmina, seguida de la lactoalbúmina y las proteínas de la carne y la soja

Page 30: Preguntas de Endocrino

CTO Medicina • C/Núñez de Balboa 115 • 28006 Madrid • Tfno.: 91 782 43 30/33/34 • E-mail: [email protected] • www. grupocto.es 30

Test 3V Endocrinología y MetabolismoCTO MEDICINA

9 ¿Cuál es el riesgo de una dieta basada exclusivamente en cereales o legumbres?

Que puede existir défi cit de ciertos aminoácidos

10

Asocia cada carencia vitamínica con la enfermedad que produce:- Vitamina B1 o tiamina- Niacina- Vitamina B6 o piridoxina- Ácido fólico- Vitamina B12

- Vitamina C- Vitamina D- Vitamina E- Vitamina K

- Beri-beri/encefalopatía de Wernicke- Pelagra: diarrea, dermatitis, demencia- Polineuropatía, dermatitis, glositis- Anemia macrocítica, trombocitopenia, leucopenia, glositis, diarrea- Anemia perniciosa, polineuropatía, glositis- Escorbuto (hiperqueratosis, sangrado gingival, petequias)- Ceguera nocturna, xeroftalmia, hiperqueratosis folicular.- Raquitismo y osteomalacia- Anemia hemolítica, retinopatía, polineuropatía- Sangrado y aumento de tiempos de coagulación

11 El exceso de vitamina A se asocia a… Pseudotumor cerebri

12 El exceso de vitamina D puede producir… Hipercalcemia

13 La vitamina K en exceso durante el embarazo puede producir Ictericia neonatal

14 El exceso de fl úor puede producir… Tinción irreversible de los dientes (fl uorosis)

15¿Cuáles son los mejores índices para valorar el estado nutricional?

El peso y sus combinaciones (IMC, % de pérdida de peso y % de pérdida de peso respecto a peso ideal)

16 ¿Cuáles son las determinaciones fundamentales para valorar la desnutrición calórica?

Las alteraciones antropométricas (peso, pliegues cutáneos y circunferencia media braquial)

17 Determinaciones que nos permiten valorar la desnutrición proteica o Kwashiorkor

El índice creatinina-altura y la determinación de proteínas como la albúmina o la transferrina

18 ¿Cuál es la vida media plasmática de la albúmina? 20 días

19¿Qué tipo de nutrición se debe instaurar, siempre que sea posible, ante un paciente desnutrido?

Nutrición enteral

20 ¿Cuál es la vía preferida para la nutrición enteral?La vía oral siempre que sea posible (no cuando la deglución es inadecuada)

21Si el tiempo de la nutrición se va a prolongar durante semanas o meses, ¿cuál será la mejor vía de administración?

Gastrostomía o yeyunostomía

22 Indicaciones de la nutrición enteral

Ingesta oral no adecuada, incapacidad para ingesta oral, fístula enterocutánea de bajo débito (<500 ml/día), pancreatitis aguda y en la resección intestinal masiva en combinación con la parenteral

23 Contraindicaciones de la nutrición enteralObstrucción intestinal, íleo, fístula de alto débito, intolerancia a las sondas

24 Complicaciones fundamentales de la nutrición enteralDiarrea, aspiración, obstrucción de la sonda, residuo gástrico y esofagitis

25Cuando un paciente va a recibir nutrición parenteral durante dos días, puede administrarse mediante una vía…

Periférica

Page 31: Preguntas de Endocrino

CTO Medicina • C/Núñez de Balboa 115 • 28006 Madrid • Tfno.: 91 782 43 30/33/34 • E-mail: [email protected] • www. grupocto.es 31

Test 3V Endocrinología y MetabolismoCTO MEDICINA

26Si el paciente va a recibir nutrición parenteral durante un tiempo prolongado es necesario administrarla mediante una vía…

Central

27 Indicaciones de nutrición parenteralMás de 7 días de reposo intestinal, tracto gastrointestinal no funcionante, no tolerancia enteral, resección intestinal masiva, fístula enterocutánea de alto débito (>500 ml/día)

28 Contraindicaciones de la nutrición parenteralRestitución de ingesta oral en menos de 7 días, vía enteral adecuada, más riesgos que benefi cios de la nutrición parenteral

29 Complicaciones de la nutrición parenteral

Aumentan los requerimientos de insulina en los diabéticos, fl ebitis en caso de vías periféricas, infecciones y otras complicaciones debidas a catéter central, alteraciones hidroelectrolíticas, esteatosis hepática y colelitiasis

30 Se considera que existe hiperlipoproteinemia cuando…El nivel de colesterol plasmático es superior a 200 mg/dl y el nivel de triglicéridos superior a 200 mg/dl

31 Las hiperlipoproteinemias más frecuentes son…… y el tratamiento inicial debe ser…

Secundarias, en relación con la obesidad, alcoholismo, mal control de la diabetes ….. el de la causa (dieta, ejercicio físico, abandonar el alcohol, control de la diabetes…)

32 ¿Cuál es la hiperlipoproteinemia primaria donde existe aumento de los niveles de HDL colesterol?

En la hiperalfaproteinemia familiar

33Varón de 30 años de edad, con antecedentes familiares de hipercolesterolemia y cifras de colesterol total de 650, ¿cuál es el diagnóstico más probable?

Hipercolesterolemia familiar monogénica

34 ¿Cuál es el tipo de dislipemia más frecuentemente asociada a la DM tipo 2?

La hipertrigliceridemia por exceso de producción de VLDL, descenso de HDL-col y aparición de partículas de LDL pequeñas

35 La prioridad en el control de la dislipemia diabética es… Control adecuado de niveles de LDL < 100

36 Cita algunos fármacos que producen hiperlipemia como efecto secundario

Estrógenos, retinoides, glucocorticoides, inhibidores de la proteasa en el tratamiento del VIH…

37¿Cuáles son las características del síndrome metabólico producido por inhibidores de la proteasa?

Hipertrigliceridemia, resistencia a la insulina, lipodistrofi a (redistribución de grasa corporal con acúmulo abdominal y retrocervical), y adelgazamiento de extremidades y de la cara

38 Tratamiento inicial de las dislipemias

Dieta, eliminar factores agravantes: control metabólico de la diabetes mellitus, control del hipotiroidismo, abstinencia de alcohol… y suspensión de otros factores de riesgo cardiovascular: HTA, tabaco…

39¿Cuándo está indicado iniciar tratamiento farmacológico en el tratamiento de las dislipemias?

Cuando no se logra disminuir sufi cientemente las cifras de colesterol y triglicéridos después de tres a seis meses de tratamiento no farmacológico intensivo

40

Une cada uno de los mecanismos de acción con el fármaco correspondiente:- Bloquean la recirculación enterohepática de los ácidos biliares y disminuyen su cantidad total- Inhibidores de la HMG CoA reductasa, enzima limitante para la síntesis de colesterol endógeno- Inhibición de la absorción intestinal de colesterol- Inhiben producción de VLDL y aumentan su depuración- Inhibe movilización de ácidos grasos desde el adipocito

- Resinas- Estatinas- Ezetimibe- Fibratos- Ácido nicotínico o niacina

Page 32: Preguntas de Endocrino

CTO Medicina • C/Núñez de Balboa 115 • 28006 Madrid • Tfno.: 91 782 43 30/33/34 • E-mail: [email protected] • www. grupocto.es 32

Test 3V Endocrinología y MetabolismoCTO MEDICINA

41 ¿Cuál es el fármaco que más aumenta el HDL-col? El ácido nicotínico

42Efecto secundario más grave de las estatinas, sobre todo en combinación con fi bratos

Riesgo de miopatía grave y rabdomiolisis

43 ¿En quiénes están contraindicadas las estatinas? Niños y embarazadas

44Enfermedad metabólica más prevalente en el mundo occidental

La obesidad

45 ¿Qué tipo de obesidad es la más relacionada con aumento de riesgo cardiovascular?

La obesidad de tipo androide

46 Principales causas de morbilidad en la obesidad

Mayor asociación a DM, HTA, dislipemia, enfermedad vascular, afectación osteoarticular, síndrome de apnea obstructiva del sueño, hernia de hiato, colelitiasis, aumento de algunos cánceres, trastornos psiquiátricos y edad ósea avanzada en niños, con adelanto puberal secundario a obesidad

47 La mayoría de los pacientes con obesidad tienen una obesi-dad de tipo

Exógena, por aumento de la ingesta calórica sin que haya desgaste

48 Defi nición de síndrome metabólico o síndrome X

- TA > 130/85 mmHg- Triglicéridos >150 mg/dl- HDL < 40 mg/dl varón / < 50 mg/dl mujer- Glucemia en ayunas > 110 mg/dl- Circunferencia de cintura > 102 varón / > 88 mujer

49 Según el IMC la obesidad se puede dividir en varios grados. Se habla de obesidad grado 4 o extrema con un IMC de…

> 50

50 Tratamiento inicial de la obesidad

- Dieta hipocalórica, equilibrada y aceptada por el paciente: 800-500 kcal- Ejercicio físico moderado y adaptado al paciente- Valorar y tratar otros factores de riesgo cardiovascular

51 Tratamiento farmacológico existente para la obesidad

- Inhibidores de la absorción (orlistat). Inhibe lipasas intestinales- Anorexígenos: fl uoxetina y derivados (útil cuando los pacientes tienen conductas bulímicas o sufren depresión), y sibutramina (disminuye ansiedad y tiene también efecto antidepresivo) - Fármacos reguladores del metabolismo energético (no han demostrado efi cacia)- Fibra dietética: para disminuir el hambre

52 Antes de iniciar tratamiento quirúrgico de la obesidad es fundamental…

Hacer una entrevista al paciente, comprobar que cumple los criterios necesarios, que no presenta ningún trastorno psiquiátrico y que comprende los riesgos derivados de la cirugía

53 Actualmente la técnica quirúrgica restrictiva que más se emplea es…

La banda gástrica ajustable

53 Actualmente la técnica quirúrgica restrictiva que más se emplea es…

La banda gástrica ajustable

54La técnica quirúrgica más empleada en la actualidad para el tratamiento de la obesidad es…

El bypass gástrico con Y de Roux

55¿Cuál es el principal inconveniente de las técnicas malabsortivas puras que se emplean en la cirugía de la obesidad?

Malnutrición

Page 33: Preguntas de Endocrino

CTO Medicina • C/Núñez de Balboa 115 • 28006 Madrid • Tfno.: 91 782 43 30/33/34 • E-mail: [email protected] • www. grupocto.es 33

Test 3V Endocrinología y MetabolismoCTO MEDICINA

56 Tanto en el preoperatorio temprano como en fases más tardías, la principal complicación de la cirugía bariátrica es…

Los vómitos

57 Objetivos principales del control de la dislipemia

- Pacientes con cardiopatía isquémica, DM o equivalentes: LDL < 100- 2 o más factores de riesgo cardiovascular: LDL < 130- 1 o ningún factor de riesgo cardiovascular: LDL < 160- HDL > 40- Triglicéridos < 200 (<150 en pacientes con DM)

TEMA 8. TRASTORNOS DEL METABOLISMO DEL CALCIO

1 Causas más comunes de hipercalcemia Enfermedad paratiroidea y enfermedades malignas

2 Causa más frecuente de hipercalcemiaHiperparatiroidismo primario por adenoma paratiroideo único o múltiple

3Causa más frecuente de hipercalcemia en pacientes ingresados

Neoplasias

4

Une cada mecanismo causante de hipercalcemia con la enfermedad correspondiente:- Hipercalcemia hipocalciúrica familiar- Intoxicación por vitamina D- Tratamiento con litio- Hiperparatiroidismos por adenoma o hiperplasia- Exceso de vitamina A- Hipertiroidismo- Tiacidas- Enfermedades granulomatosas

- Exceso de PTH- Exceso de vitamina D- Exceso de PTH- Exceso de PTH- Aumento del recambio óseo- Aumento del recambio óseo- Aumento de la reabsorción de calcio en el riñón- Exceso de vitamina D

5

Une cada una de las siguientes neoplasias con el mecanismo implicado en la hipercalcemia:- Destrucción ósea local- Producción de PTHrp- Producción de vitamina D

- Mieloma múltiple, cáncer de mama- Carcinoma epidermoide de pulmón, carcinoma renal- Linfoma B

6 Clínica típica de la hipercalcemia

Cansancio, depresión, confusión, letargia, anorexia, estreñimiento, náuseas, vómitos, aumento de la diuresis, acortamiento del QT, arritmias y calcifi caciones ectópicas en vasos, articulaciones, córnea…

7Paciente de 60 años con urolitiasis, poliuria, úlcera péptica y osteopenia generalizada con antecedente de fractura de cúbito y radio, ¿qué habría que sospechar?

Hiperparatiroidismo primario

8Manifestaciones clínicas más típicas del hiperparatiroidismo primario

- Enfermedad ósea: osteítis fi brosa quística (grave) u osteopenia (más frecuente)- Enfermedad renal: nefrolitiasis y nefrocalcinosis

9 Paciente de 50 años con calcemia de 12 mg/dl asintomático y fumador, ¿cuál es la etiología más probable?

Carcinoma pulmonar con hipercalcemia

10Mujer de 39 años con crisis renoureterales de repetición y niveles de calcio y PTH elevados, ¿qué método de localización de la lesión hay que realizar preoperatoriamente?

Ninguno

11Datos bioquímicos más frecuentes en el hiperparatiroidismo primario

Hipercalcemia, hipofosfatemia, PTH elevada, acidosis metabólica, aumento de vitamina D y en orina: hipercalciuria y aumento reabsorción tubular de fosfatos

Page 34: Preguntas de Endocrino

CTO Medicina • C/Núñez de Balboa 115 • 28006 Madrid • Tfno.: 91 782 43 30/33/34 • E-mail: [email protected] • www. grupocto.es 34

Test 3V Endocrinología y MetabolismoCTO MEDICINA

TEMA 8. TRASTORNOS DEL METABOLISMO DEL CALCIO

12Prueba de imagen más rentable para el diagnóstico de localización de hiperparatiroidismo primario

Ecografía cervical más gammagrafía con Tc-sesta-mibi

13Paciente de 70 años que acude a urgencias por confusión mental, náuseas, vómitos, estreñimiento y calcemia de 15 mg/dl, ¿cuál será el tratamiento más adecuado?

Hidratación con suero salino y furosemida iv

14 Indicaciones de cirugía en el hiperparatiroidismo primario

- Menores de 50 años- Mayores de 50 años si: * Calcio sérico mayor de 11,5 mg/dl * Historia de episodio previo de hipercalcemia potencialmente letal, enfermedad neuromuscular grave o fracturas * Aclaramiento de creatinina <60 ml/min * Cálculos renales en la radiografía de abdomen o nefrolitiasis recidivante * Reducción de masa ósea más de 2,5 desviaciones típicas en la densitometría

15 Consecuencia más grave tras el tratamiento quirúrgico del hiperparatiroidismo primario

Hipocalcemia postoperatoria

16

Paciente intervenido de hiperplasia paratiroidea hace 24 horas que presenta calcio menor a 8 mg/dl con signos de Trousseau y Chvostek positivos, ¿cuál será el tratamiento más adecuado?

Tratamiento con calcio parenteral

17Tipo de herencia que existe en la hipercalcemia hipocalciúrica familiar

Autosómica dominante

18Clínica más frecuente de la hipercalcemia hipocalciúrica familiar

Asintomática

19

Niño de 7 años de edad, asintomático, en el que se detecta una calcemia de 11,2 mg/dl, hipermagnesemia e hipocalciuria. PTH elevada y vitamina D normal. Actitud más adecuada

Vigilancia

20¿Cuáles son los tumores más frecuentemente asociados a hipercalcemia?

Los epidermoides

21

Paciente que ingresa en urgencias con insufi ciencia renal, hiperfosfatemia y calcemia de 14 mg/dl, náuseas, vómitos, prurito y calcifi caciones ectópicas vasculares, ¿cuál es el tratamiento más adecuado?

Hidratación con suero salino y diálisis

22 Tratamiento médico fundamental del hiperparatiroidismo secundario asociado a insufi ciencia renal

Reducción de la ingesta de fosfato de la dieta, administración de hidróxido de aluminio y calcitriol

23 ¿Cuándo están indicados los glucocorticoides en la hipercalcemia?

En la hipercalcemia tumoral, sobre todo en los tumores hematológicos, y en la hipercalcemia por exceso de vitamina D

24Tratamientos más utilizados en el control agudo de la hipercalcemia

Hidratación, bifosfonatos, calcitonina y furosemida

25 Tratamiento crónico de la hipercalcemiaRestricción del calcio de la dieta, hidratación oral agresiva y bifosfonatos

Page 35: Preguntas de Endocrino

CTO Medicina • C/Núñez de Balboa 115 • 28006 Madrid • Tfno.: 91 782 43 30/33/34 • E-mail: [email protected] • www. grupocto.es 35

Test 3V Endocrinología y MetabolismoCTO MEDICINA

TEMA 8. TRASTORNOS DEL METABOLISMO DEL CALCIO

26 Indicaciones de paratiroidectomía en la insufi ciencia renal crónica

- Hipercalcemia severa- Enfermedad ósea severa- Prurito que no responde a tratamiento médico ni diálisis- Calcifi caciones extraesqueléticas- Miopatía sintomática no explicada por otras causas

27En un paciente que ingresa con espasmos musculares, irritabilidad, intervalo QT prolongado y onda T invertida, ¿qué determinación será fundamental realizar?

Calcemia

28 Causa más frecuente de défi cit de PTH Hipoparatiroidismo postquirúrgico

29Paciente con hipertelorismo, baja implantación de las orejas, infecciones de repetición, malformaciones vasculares e hipocalcemia, ¿cuál es su sospecha diagnóstica fundamental?

Síndrome de Di George

30 Dato fundamental que diferencia el hipoparatiroidismo del pseudohipoparatiroidismo

En el hipoparatiroidismo la PTH está disminuida y en el pseudohipoparatiroidismo la PTH está elevada

31Cuadro clínico del pseudohipoparatiroidismo tipo a u osteodistrofi a hereditaria de Albright

Estatura baja, obesidad, pseudopterigium colli y acortamiento del 4º y 5º metacarpianos

32 Causas más frecuentes de hipomagnesemiaAlcoholismo crónico con ingesta nutricional pobre, malabsorción intestinal y nutrición parenteral sin aporte de magnesio

33

En un paciente alcohólico con desorientación, obnubilación, mioclonías y fasciculaciones, niveles de potasio, calcio y magnesio bajos, ¿qué elemento es el más necesario de aportar para corregir todo el trastorno?

Magnesio

34 Paciente con hipocalcemia, hipomagnesemia e hipofosfatemia, ¿qué es lo más probable?

Malabsorción intestinal

35 Cuadros que cursan con hipocalcemia e hiperfosfatemiaHipoparatiroidismo, pseudohipoparatiroidismo y en la insufi ciencia renal inicialmente

36 Cuadro bioquímico típico del hipoparatiroidismoHipocalcemia, hiperfosfatemia, PTH disminuida y en orina: hipercalciuria y aumento de la reabsorción de fosfato

37 Tratamiento del hipoparatiroidismoVitamina D más suplementos de calcio. También diuréticos tiacídicos para disminuir la eliminación urinaria de calcio

TEMA 9. TRASTORNOS QUE AFECTAN A SISTEMAS ENDOCRINOS MÚLTIPLES

1Manifestación clínica más frecuente del MEN1 o síndrome de Wermer

Hiperparatiroidismo primario

2 Causa más frecuente de hiperparatiroidismo primario en un MEN1

Hiperplasia paratiroidea

3 Causa más frecuente de hiperparatiroidismo primario aislado Adenoma único

4 Ordena de más frecuente a menos frecuente las manifestaciones clínicas del MEN1

Hiperparatiroidismo, neoplasia de los islotes pancreáticos y tumor hipofi sario

5 Tumores hipofi sarios más frecuentes en el seno de un MEN1 Prolactinoma y adenoma productor de GH

Page 36: Preguntas de Endocrino

CTO Medicina • C/Núñez de Balboa 115 • 28006 Madrid • Tfno.: 91 782 43 30/33/34 • E-mail: [email protected] • www. grupocto.es 36

Test 3V Endocrinología y MetabolismoCTO MEDICINA

TEMA 9. TRASTORNOS QUE AFECTAN A SISTEMAS ENDOCRINOS MÚLTIPLES

6 Tipo de herencia en los síndromes MEN Autosómica dominante

7 Tipo de gen alterado en el MEN1Inactivación de un gen supresor de tumores localizado en cromosoma 11

8 Tipo de gen alterado en el MEN2 Mutación de protooncogén RET localizado en cromosoma 10

9 Tipo de MEN en que está indicado el estudio genético de forma universal

MEN2

10Paciente con carcinoma medular de tiroides y feocromocitoma, ¿qué síndrome es el más probable?

MEN2

11

Paciente de 27 años de edad, diagnosticado hace dos meses de feocromocitoma, al que se le descubre un nódulo tiroideo frío en la gammagrafía y aumento de la calcitonina, ¿qué se debe sospechar?

Carcinoma medular de tiroides

12 Manifestaciones clínicas del MEN2ACarcinoma medular de tiroides, feocromocitoma e hiperparatiroidismo

13Paciente con catecolaminas elevadas en sangre periférica, neuromas mucosos, hábito marfanoide y nódulo tiroideo frío en gammagrafía, ¿cuál es el diagnóstico más probable?

MEN2B

14

Paciente de 40 años intervenida por carcinoma medular de tiroides en el seno de un MEN2A, ¿cuál es el método para identifi car a familiares de esta paciente con riesgo de desarrollar carcinoma medular?

Estudio genético en células de sangre periférica

15En el estudio uno de sus hijos presenta mutación en el protooncogén RET, ¿cuál es la actitud más adecuada?

Tiroidectomía precoz

16 Características del feocromocitoma en el seno de un MEN2AMás frecuentemente bilateral y productor de adrenalina más que de noradrenalina

17Paciente con carcinoma medular de tiroides y feocromocitoma en el seno de un MEN2, ¿qué tumor intervendrías primero?

El feocromocitoma

18 Rasgo más característico del MEN2B: neuromas de las mucosas En la punta de la lengua, párpados y tubo digestivo

19 Tipo de herencia en síndromes poliglandulares autoinmunes- PGA tipo 1: autosómica recesiva, gen AIRE en cromosoma 21- PGA tipo 2: herencia poligénica, asociado a haplotipos HLA DR3 y DR4

20Tríada característica del síndrome poliglandular autoinmune tipo 1

- Candidiasis mucocutánea- Hipoparatiroidismo- Insufi ciencia suprarrenal

21 Niña de 8 años de edad con candidiasis mucocutánea, diabetes mellitus e hipogonadismo, ¿cuál es la sospecha?

Síndrome poliglandular autoinmune tipo 1

22 Manifestación clínica más frecuente del síndrome poliglandular autoinmune tipo 2 o síndrome de Schmidt

La insufi ciencia suprarrenal primaria

Page 37: Preguntas de Endocrino

CTO Medicina • C/Núñez de Balboa 115 • 28006 Madrid • Tfno.: 91 782 43 30/33/34 • E-mail: [email protected] • www. grupocto.es 37

Test 3V Endocrinología y MetabolismoCTO MEDICINA

23Paciente de 36 años con diabetes mellitus, enfermedad de Addison y amenorrea de 4 meses de evolución con niveles de LH y FSH elevados y estradiol bajo. Diagnóstico más probable

Síndrome poliglandular autoinmune tipo 2

24Tríada más característica del síndrome poliglandular autoinmune tipo 2

- Adrenalitis autoinmune- Enfermedad tiroidea autoinmune- Diabetes mellitus tipo 1

TEMA 10. TUMORES DE LAS CÉLULAS DE LOS ISLOTES PANCREÁTICOS

1 Clínica fundamental para el diagnóstico de vipomaDiarrea secretora con volumen de heces mayor de 3 l/día, hipopotasemia e hipoclorhidia

2 Diagnóstico de vipomaAumento de la concentración de VIP en plasma con volumen de heces de al menos 1 litro al día

3 Tratamiento fundamental del vipoma Cirugía y tratamiento de sostén con fl uidoterapia y electrolitos

4 Si existen metástasis Quimioterapia

5Tumor cuya clínica es conocida como síndrome de las 4D (Diabetes, Depresión, Dermatitis y Deep vein trombosis)

Glucagonoma

6 Afectación cutánea típica del glucagonoma Eritema necrolítico migratorio

7 Diagnóstico de glucagonoma Glucagón elevado en ayunas y que no se suprime con glucosa

8 Tríada clínica clásica del somatostatinoma Diabetes, diarrea-esteatorrea y colelitiasis

9 Diagnóstico del somatostatinoma Niveles de somatostatina plasmática elevados

10Paciente de 60 años de edad con diarrea, esteatorrea, úlcera gástrica que ha sangrado repetidas veces, ¿qué prueba le realizaría?

Medición de niveles de gastrina

11Se obtiene una gastrina de 300 pg/ml en presencia de hipersecreción gástrica, ¿qué tratamiento sería el más adecuado?

Tratamiento quirúrgico del tumor y sintomático con anti-H2 y omeprazol

12 Clínica característica de tumores pancreáticos no funcionantes Por compresión: ictericia, varices esofágicas, dolor abdominal

13 Tratamiento de los tumores no funcionantesTratamiento quirúrgico curativo en menos del 20% de casos y escasa respuesta a quimioterapia (mal pronóstico)

14 Tumores endocrinos más frecuentes del aparato digestivo Tumores carcinoides

15 Localización más frecuente de los tumores carcinoides Apéndice, recto e íleon

16 Ordena de mayor a menor grado de malignidad las siguientes localizaciones en los tumores carcinoides

Apéndice, duodeno y recto: duodeno (más malignidad), recto y apéndice

17¿Cuál es la localización de los tumores carcionoides que pueden producir síndrome carcinoide sin que existan metástasis hepáticas?

Los extraintestinales y pulmonares

18Tipo de carcinoide de localización intestinal en que son más frecuentes las metástasis hepáticas y el síndrome carcinoide

En los de intestino delgado

Page 38: Preguntas de Endocrino

CTO Medicina • C/Núñez de Balboa 115 • 28006 Madrid • Tfno.: 91 782 43 30/33/34 • E-mail: [email protected] • www. grupocto.es 38

Test 3V Endocrinología y MetabolismoCTO MEDICINA

TEMA 10. TUMORES DE LAS CÉLULAS DE LOS ISLOTES PANCREÁTICOS

19 Producto que secretan con más frecuencia los carcinoides 5-hidroxitriptamina

20

Paciente de 60 años con historia de diarrea y pérdida de peso junto con crisis súbitas de enrojecimiento facial que se acentúan con la ingesta de alcohol y telangiectasias sobre mejillas. Se diagnostica como carcionoide intestinal, ¿qué sospecharías que apareciera en el estudio de extensión?

Metástasis hepáticas

21 Clínica característica del síndrome carcinoideSíndrome debilitante con rubefacción cutánea, diarrea y cardiopatía valvular (sobre todo insufi ciencia tricuspídea)

22 Factores favorecedores que pueden desencadenar la clínica de síndrome carcinoide

Estrés, ingesta de alimentos, alcohol

23Prueba diagnóstica de mayor utilidad para el diagnóstico de tumor carcinoide

Determinación de 5-hidroxiindolacético en orina

24 Tratamiento de elección en la mayor parte de tumores de los islotes pancreáticos

Quirúrgico

25 Tratamiento médico que puede controlar los síntomas del síndrome carcinoide

Los análogos de la somatostatina, octreótido y lanreótido

26Cita dos ejemplos de tumores que pueden producir síndrome carcinoide sin metástasis hepáticas

Carcinoide bronquial y ovárico

27Paciente al que se le realiza una apendicectomía y se descubre un carcinoide de 1 centímetro de diámetro que llega hasta submucosa, ¿qué actitud es la más adecuada?

Revisiones periódicas

TEMA 11. TRASTORNOS DE LA DIFERENCIACIÓN Y EL DESARROLLO CELULAR

1Défi cit enzimático más frecuente en la hiperplasia suprarrenal congénita

Défi cit de 21-hidroxilasa

2

Varón que a los 7 días de su nacimiento comienza con un cuadro de vómitos y deshidratación. Concentración sérica de sodio: 120 mEq/l y potasio 9 mEq/l, ¿cuál es el diagnóstico más probable?

Hiperplasia suprarrenal congénita

3 Diagnóstico del défi cit de 21-hidroxilasa Elevación en sangre de 17-hidroxiprogesterona

4 Cuadro clínico clásico del défi cit de 21-hidroxilasa en un recién nacido

Hiponatremia, hiperpotasemia, vómitos, anorexia, depleción de volumen, shock hipovolémico

5 Defectos hormonales que existen en un síndrome pierde sal por défi cit de 21-hidroxilasa severo

Défi cit de aldosterona y cortisol

6Alteración crosómica más frecuente que afecta a los cromoso-mas sexuales

Síndrome de Klinefelter

7Alteración cromosómica más frecuente en el síndrome de Klinefelter

47XXY

8 Manifestaciones típicas del síndrome de KlinefelterTalla alta, testículos pequeños, ginecomastia e hipogonadismo primario

Page 39: Preguntas de Endocrino

CTO Medicina • C/Núñez de Balboa 115 • 28006 Madrid • Tfno.: 91 782 43 30/33/34 • E-mail: [email protected] • www. grupocto.es 39

Test 3V Endocrinología y MetabolismoCTO MEDICINA

TEMA 11. TRASTORNOS DE LA DIFERENCIACIÓN Y EL DESARROLLO CELULAR

9 Manifestaciones clínicas típicas del síndrome de Turner Talla baja, linfedema, pterigium colli

10Niña de 6 años de edad, coartación de aorta, talla baja, infantilismo sexual y pterigium colli, diagnóstico más probable y exploración para confi rmar el diagnóstico

Síndrome de Turner y cariotipo

11

Varón, 18 años de edad, 180 cm de altura, distribución ginecoide de la grasa, ausencia de vello facial, ginecomastia y testículos de 1,5 cm. LH y FSH elevadas y azoospermia. ¿Cuál será la prueba fundamental para el diagnóstico? … y ¿qué cariotipo es de esperar?

Cariotipo… 47XXY

12Alteración de los cromosomas sexuales que se asocia más frecuentemente con coartación de aorta e hipotiroidismo autoinmune

Síndrome de Turner

13 Causa más frecuente de amenorrea primaria Síndrome de Turner

14 Tratamiento del síndrome de TurnerFeminización con estrógenos en dosis crecientes hasta desarrollo mamario, GH y oxandrolona para aumentar la talla fi nal

15 Cariotipo de la disgenesia gonadal pura 46XX o 46XY

16 Los genitales externos en la disgenesia gonadal pura son… Genitales femeninos inmaduros

17 Los genitales internos son… Útero y trompa de Falopio infantiles

18 Las gónadas son… Cintillas bilaterales

19El tratamiento fundamental de la disgenesia gonadal pura es…

Extirpar las cintillas gonadales por la elevada incidencia de tumores

20 Causa más frecuente de genitales ambiguos en el recién nacido

Défi cit de 21-hidroxilasa

21 Segunda causa más frecuente de ambigüedad genital Disgenesia gonadal mixta

22Tumor gonadal del testículo abdominal más frecuente en la disgenesia gonadal mixta

Seminoma

23 Característica fundamental del hermafroditismo verdaderoExistencia de epitelio ovárico y testicular, 1 ovario y 1 testículo o un ovoteste

24 Tratamiento más importante del hermafroditismo verdaderoElegir sexo en la infancia y extirpar la gónada y conductos que no correspondan con el sexo asignado (2/3 se desarrollan como varones)

25Recién nacida de 21 días de vida con hipertrofi a de clítoris, fusión labioescrotal y virilización de la uretra, ¿cuál es la causa más frecuente de estas alteraciones?

Défi cit de 21-hidroxilasa

26 Consecuencia fundamental sobre el esqueleto del défi cit de 21-hidroxilasa

Talla corta

27 Clínica característica de un defecto de 21-hidroxilasa leve y de aparición tardía

Virilización, hirsutismo y trastornos menstruales

Page 40: Preguntas de Endocrino

CTO Medicina • C/Núñez de Balboa 115 • 28006 Madrid • Tfno.: 91 782 43 30/33/34 • E-mail: [email protected] • www. grupocto.es 40

Test 3V Endocrinología y MetabolismoCTO MEDICINA

TEMA 11. TRASTORNOS DE LA DIFERENCIACIÓN Y EL DESARROLLO CELULAR

28 Principal esteroide secretado en el défi cit de 21-hidroxilasa 17-hidroxiprogesterona

29 ¿Para el diagnóstico de qué enfermedad se emplea la respuesta de 17-hidroxiprogesterona al estímulo con ACTH?

Para el diagnóstico de pseudohermafroditismo femenino por défi cit de 21-hidroxilasa

30 Característica clínica principal que diferencia el défi cit de 21-hidroxilasa del défi cit de 11-hidroxilasa

- Défi cit de 21-hidroxilasa: hipotensión por síndrome pierde sal- Défi cit de 11-hidroxilasa: hipertensión por aumento de desoxicorticosterona (DOCA)

31 Diagnóstico del défi cit de 11-hidroxilasa Respuesta de 11-desoxicortisol al estímulo con ACTH

32 Segunda causa más frecuente de síndrome pierde sal Défi cit de 3-beta-hidroxiesteroide deshidrogenasa

33 Diferencia clínica fundamental del défi cit de 3-beta-hidroxiesteroide deshidrogenasa con respecto a los anteriores

Virilización leve en la mujer y ausencia de virilización y ginecomastia en el varón

34 Principal esteroide excretado en el défi cit enzimático anterior Dehidroepiandrosterona (DHEA)

35Características del síndrome de Mayer-Rokitansky-Kuster-Hauser

Ausencia de vagina asociada a útero hipoplásico o ausente

36 Diagnóstico del síndrome anterior En la pubertad, por falta de menstruación

37 Tratamiento fundamental Creación de neovagina

38 Causas de pseudohermafroditismo masculino- Anomalías en la síntesis de andrógenos- Anomalías en la acción de los andrógenos- Síndrome de persistencia del conducto mülleriano

39

Niño que fallece a los 2 años de edad, con virilización incompleta, genitales ambiguos, y en la autopsia se observan suprarrenales aumentadas de tamaño y cargadas de lípidos, ¿cuál es la causa más probable?

Défi cit de StAR (proteína reguladora de la esteroidogénesis), también llamado hiperplasia suprarrenal congénita lipoide

40 El défi cit de cortisol, aldosterona y andrógenos es propio de…. y se caracteriza clínicamente por…

Défi cit de 17-hidroxilasa ….en varones, pseudohermafroditismo; y en mujeres, falta de feminización e infantilismo, además de hipertensión e hipopotasemia

41 Tratamiento de la hiperplasia suprarrenalTratamiento sustitutivo con glucocorticoides y, a veces, mineralcorticoides

42 Tratamiento de recién nacida hembra con genitales ambiguos Feminización con estrógenos en pubertad

43 Recién nacido varón con genitales ambiguos, ¿qué actitud adoptarías?

Si existe defecto anatómico importante: desarrollo como hembra con corrección quirúrgica, extirpación de gónadas y feminización

44Mujer con amenorrea, genitales externos femeninos, escasez de vello axilar y pubiano, mama femenina, clítoris normal y testículos abdominales; diagnóstico más probable

Síndrome de Morris

45 Características hormonales del síndrome de MorrisTestosterona elevada, estrógenos elevados y LH elevada, con resistencia androgénica completa por alteración del receptor androgénico

Page 41: Preguntas de Endocrino

CTO Medicina • C/Núñez de Balboa 115 • 28006 Madrid • Tfno.: 91 782 43 30/33/34 • E-mail: [email protected] • www. grupocto.es 41

Test 3V Endocrinología y MetabolismoCTO MEDICINA

TEMA 11. TRASTORNOS DE LA DIFERENCIACIÓN Y EL DESARROLLO CELULAR

46 Principal tratamiento del síndrome de MorrisExtirpar gónadas abdominales o inguinales por posible desarrollo de tumores

47 Primar signo de desarrollo puberal en niñas Aparición de botón mamario

48 Primer signo de desarrollo puberal en niños Aumento del volumen testicular

49 ¿A qué se denomina pubertad precoz?- Desarrollo sexual en niños antes de los 9 años- Inicio de botón mamario antes de los 8 años en niñas o menarquia antes de los 9 años

50 ¿A qué se llama precocidad isosexual?Cuando el desarrollo prematuro es adecuado para sexo cromosómico y gonadal

51 ¿A qué se llama precocidad heterosexual?Cuando el desarrollo prematuro no coincide con el sexo genético, es decir, cuando se da feminización en varones y virilización en mujeres

52Causas más frecuentes de pubertad precoz verdadera en niñas

- Idiopática en el 90%- Enfermedad orgánica cerebral, como tumores, en el 10% de casos- Hiperplasia suprarrenal congénita virilizante

53 Cuadros de pubertad precoz incompleta en niñasTelarquia prematura aislada, adrenarquia y pubarquia precoz idiopáticas

54Niña de 7 años de edad con inicio de botón mamario y vello axilar, ¿cuándo será de esperar que ocurra su desarrollo puberal?

De manera normal, en el momento esperado y alcanzando una talla normal

55 ¿A qué está predispuesta esta niña en un futuro?Al riesgo de hiperinsulinismo y aumento de riesgo de hiperandrogenismo ovárico

56 ¿A qué se refi ere el término pseudopubertad precoz?

- En varones: virilización por aumento de testosterona que no se acompaña de espermatogénesis- En mujeres: feminización por aumento de secreción de estrógenos pero sin ciclos ovulatorios

57 Causas más frecuentes

- En varones: tumores de células de Leydig, tumores secretores de hCG, tumores suprarrenales o hiperplasia suprarrenal congénita- En mujeres: quistes y tumores ováricos (tumores de células de la granulosa-teca)

58 Causas de virilización en niñas antes de la pubertad (pubertad precoz heterosexual)

Hiperplasia suprarrenal congénita, secreción de andrógenos por tumores ováricos o suprarrenales

59 Causas de feminización en niños antes de la pubertadAumento de estrógenos por tumores testiculares, hermafroditismo verdadero, carcinoma de pulmón, tumores productores de hCG, fármacos…

60Tipo de población en la que es más frecuente la pubertad precoz

Mujeres

61Tipo de población en la que es más frecuente el retraso constitucional del crecimiento

Varones

Page 42: Preguntas de Endocrino

CTO Medicina • C/Núñez de Balboa 115 • 28006 Madrid • Tfno.: 91 782 43 30/33/34 • E-mail: [email protected] • www. grupocto.es 42

Test 3V Endocrinología y MetabolismoCTO MEDICINA

TEMA 11. TRASTORNOS DE LA DIFERENCIACIÓN Y EL DESARROLLO CELULAR

62 ¿A qué denominamos retraso de la adolescencia?Al comienzo de la pubertad a una edad posterior a la media (después de los 16 años)

63Hallazgos hormonales en paciente varón con pubertad retrasada

Disminución de testosterona y LH y FSH disminuidas

64Hallazgos hormonales en paciente con ausencia de desarrollo puberal por hipogonadismo primario

FSH y LH elevadas y testosterona disminuida